Chapter 29: Nursing Management: Obstructive Pulmonary Diseases, Respiratory Failure and Acute Respiratory Distress Syndrome, Ch 31 Upper Respiratory System, Ch 32 Lower Respiratory System, Chapter 40: Respiratory Dysfunction, Chapter 23

¡Supera tus tareas y exámenes ahora con Quizwiz!

When describing the drugs used to treat lower respiratory system conditions, the nursing instructor discusses chronic obstructive pulmonary disease (COPD). Which condition would the instructor include when discussing this condition related to the use of these drugs? Select all that apply. A) Asthma B) Pulmonary embolism C) Pulmonary hypertension D) Chronic bronchitis E) Pneumonia

@Ans: A, D Feedback: COPD encompasses asthma, chronic bronchitis, chronic obstructive bronchitis, emphysema, or a combination of the conditions.

A patient with asthma has a personal best peak expiratory flow rate (PEFR) of 400 L/minute. When explaining the asthma action plan, the nurse will teach the patient that a change in therapy is needed when the PEFR is less than ___ L/minute

ANS: 320 A PEFR less than 80% of the personal best indicates that the patient is in the yellow zone where changes in therapy are needed to prevent progression of the airway narrowing. DIF: Cognitive Level: Apply (application) REF: 579 TOP: Nursing Process: Evaluation MSC: NCLEX: Physiological Integrity

An 18-month-old child is seen in the clinic with AOM. Trimethoprim-sulfamethoxazole (Bactrim) is prescribed. Which statement made by the parent indicates a correct understanding of the instructions? a. I should administer all the prescribed medication. b. I should continue medication until the symptoms subside. c. I will immediately stop giving medication if I notice a change in hearing. d. I will stop giving medication if fever is still present in 24 hours.

ANS: A Antibiotics should be given for their full course to prevent recurrence of infection with resistant bacteria. Symptoms may subside before the full course is given. Hearing loss is a complication of AOM. Antibiotics should continue to be given. Medication may take 24 to 48 hours to make symptoms subside. It should be continued. PTS: 1 DIF: Cognitive Level: Application REF: 1206 OBJ: Nursing Process: Implementation MSC:

A school-age child has had an upper respiratory tract infection for several days and then began having a persistent dry, hacking cough that was worse at night. The cough has become productive in the past 24 hours. This is most suggestive of: a. Bronchitis. b. Bronchiolitis. c. Viral-induced asthma. d. Acute spasmodic laryngitis.

ANS: A Bronchitis is characterized by these symptoms and occurs in children older than 6 years. Bronchiolitis is rare in children older than 2 years. Asthma is a chronic inflammation of the airways that may be exacerbated by a virus. Acute spasmodic laryngitis occurs in children between 3 months and 3 years.

Which statement expresses accurately the genetic implications of cystic fibrosis (CF)? a. If it is present in a child, both parents are carriers of this defective gene. b. It is inherited as an autosomal dominant trait. c. It is a genetic defect found primarily in non-Caucasian population groups. d. There is a 50% chance that siblings of an affected child also will be affected.

ANS: A CF is an autosomal recessive gene inherited from both parents and is found primarily in Caucasian populations An autosomal recessive inheritance pattern means that there is a 25% chance that a sibling will be infected but a 50% chance a sibling will be a carrier. PTS: 1 DIF: Cognitive Level: Comprehension REF: 1235 OBJ: Nursing Process: Assessment MSC:

In providing nourishment for a child with cystic fibrosis (CF), which factor should the nurse keep in mind? a. Diet should be high in carbohydrates and protein. b. Diet should be high in easily digested carbohydrates and fats. c. Most fruits and vegetables are not well tolerated. d. Fats and proteins must be greatly curtailed.

ANS: A Children with CF require a well-balanced, high-protein, high-calorie diet because of impaired intestinal absorption. Enzyme supplementation helps digest foods; other modifications are not necessary. A well- balanced diet containing fruits and vegetables is important. Fats and proteins are a necessary part of a well- balanced diet.

A child has a chronic, nonproductive cough and diffuse wheezing during the expiratory phase of respiration. This suggests: a. Asthma. b. Pneumonia. c. Bronchiolitis. d. Foreign body in the trachea.

ANS: A Children with asthma usually have these chronic symptoms. Pneumonia appears with an acute onset and fever and general malaise. Bronchiolitis is an acute condition caused by respiratory syncytial virus. Foreign body in the trachea will manifest with acute respiratory distress or failure and maybe stridor.

A nurse is interpreting the results of a tuberculin skin test (TST) on an adolescent who is human immunodeficiency virus (HIV) positive. Which induration size indicates a positive result for this child 48 to 72 hours after the test? a. 5 mm b. 10 mm c. 15 mm d. 20 mm

ANS: A Clinical evidence of a positive TST in children receiving immunosuppressive therapy, including immunosuppressive doses of steroids, or who have immunosuppressive conditions, including HIV infection, is an induration of 5 mm. Children younger than 4 years of age (a) with other medical risk conditions, including Hodgkins disease, lymphoma, diabetes mellitus, chronic renal failure, or malnutrition; (b) born or whose parents were born in high-prevalence tuberculosis (TB) regions of the world; (c) frequently exposed to adults who are HIV infected, homeless, users of illicit drugs, residents of nursing homes, incarcerated or institutionalized, or migrant farm workers; and (d) who travel to high-prevalence TB regions of the world are positive when the induration is 10 mm. Children 4 years of age or older without any risk factors are positive when the induration is 20 mm.

An infants parents ask the nurse about preventing otitis media (OM). What should the nurse recommend? a. Avoid tobacco smoke. b. Use nasal decongestant. c. Avoid children with OM. d. Bottle-feed or breastfeed in supine position.

ANS: A Eliminating tobacco smoke from the childs environment is essential for preventing OM and other common childhood illnesses. Nasal decongestants are not useful in preventing OM. Children with uncomplicated OM are not contagious unless they show other upper respiratory infection symptoms. Children should be fed in an upright position to prevent OM.

An appropriate nursing intervention when caring for a child with pneumonia is to: a. Encourage rest. b. Encourage the child to lie on the unaffected side. c. Administer analgesics. d. Place the child in the Trendelenburg position.

ANS: A Encouraging rest by clustering care and promoting a quiet environment is the best intervention for a child with pneumonia. Lying on the affected side may promote comfort by splinting the chest and reducing pleural rubbing. Analgesics are not indicated. Children should be placed in a semi-erect position or position of comfort.

A nurse is conducting an in-service on asthma. Which statement is the most descriptive of bronchial asthma? a. There is heightened airway reactivity. b. There is decreased resistance in the airway. c. The single cause of asthma is an allergic hypersensitivity. d. It is inherited.

ANS: A In bronchial asthma, spasm of the smooth muscle of the bronchi and bronchioles causes constriction, producing impaired respiratory function. In bronchial asthma, there is increased resistance in the airway. There are multiple causes of asthma, including allergens, irritants, exercise, cold air, infections, medications, medical conditions, and endocrine factors. Atopy or development of an immunoglobulin E (IgE)mediated response is inherited but is not the only cause of asthma.

Abdominal thrusts (the Heimlich maneuver) are recommended for airway obstruction in children older than: a. 1 year. b. 4 years. c. 8 years. d. 12 years.

ANS: A The Heimlich maneuver is recommended for airway obstruction in children older than 1 year. In children younger than 1 year, back blows and chest thrusts are administered.

The earliest recognizable clinical manifestation of cystic fibrosis (CF) is: a. Meconium ileus. b. History of poor intestinal absorption. c. Foul-smelling, frothy, greasy stools. d. Recurrent pneumonia and lung infections.

ANS: A The earliest clinical manifestation of CF is a meconium ileus, which is found in about 10% of children with CF. Clinical manifestations include abdominal distention, vomiting, failure to pass stools, and rapid development of dehydration. History of malabsorption is a later sign that manifests as failure to thrive. Foul- smelling stools and recurrent respiratory infections are later manifestations of CF.

Decongestant nose drops are recommended for a 10-month-old infant with an upper respiratory tract infection. Instructions for nose drops should include: a. Avoiding use for more than 3 days. b. Keeping drops to use again for nasal congestion. c. Administering drops until nasal congestion subsides. d. Administering drops after feedings and at bedtime.

ANS: A Vasoconstrictive nose drops such as Neo-Synephrine should not be used for more than 3 days to avoid rebound congestion. Drops should be discarded after one illness because they may become contaminated with bacteria. Vasoconstrictive nose drops can have a rebound effect after 3 days of use. Drops administered before feedings are more helpful. PTS: 1 DIF: Cognitive Level: Comprehension REF: 1197 OBJ: Nursing Process: Implementation MSC:

A patient in the clinic with cystic fibrosis (CF) reports increased sweating and weakness during the summer months. Which action by the nurse would be most appropriate? a. Have the patient add dietary salt to meals. b. Teach the patient about the signs of hypoglycemia. c. Suggest decreasing intake of dietary fat and calories. d. Instruct the patient about pancreatic enzyme replacements.

ANS: A Added dietary salt is indicated whenever sweating is excessive, such as during hot weather, when fever is present, or from intense physical activity. The management of pancreatic insufficiency includes pancreatic enzyme replacement of lipase, protease, and amylase (e.g., Pancreaze, Creon, Ultresa, Zenpep) administered before each meal and snack. This patient is at risk for hyponatremia based on reported symptoms. Adequate intake of fat, calories, protein, and vitamins is important. Fat-soluble vitamins (vitamins A, D, E, and K) must be supplemented because they are malabsorbed. Use of caloric supplements improves nutritional status. Hyperglycemia due to pancreatic insufficiency is more likely to occur than hypoglycemia.DIF:

The nurse reviews the medication administration record (MAR) for a patient having an acute asthma attack. Which medication should the nurse administer first? a. Albuterol (Ventolin) 2.5 mg per nebulizer b. Methylprednisolone (Solu-Medrol) 60 mg IV c. Salmeterol (Serevent) 50 mcg per dry-powder inhaler (DPI) d. Triamcinolone (Azmacort) 2 puffs per metered-dose inhaler (MDI)

ANS: A Albuterol is a rapidly acting bronchodilator and is the first-line medication to reverse airway narrowing in acute asthma attacks. The other medications work more slowly. DIF: Cognitive Level: Apply (application) REF: 570 | 576 OBJ: Special Questions: Prioritization TOP: Nursing Process: Implementation MSC: NCLEX: Physiological Integrity

A patient who is experiencing an acute asthma attack is admitted to the emergency department. Which assessment should the nurse complete first? a. Listen to the patient's breath sounds. b. Ask about inhaled corticosteroid use. c. Determine when the dyspnea started. d. Obtain the forced expiratory volume (FEV) flow rate.

ANS: A Assessment of the patient's breath sounds will help determine how effectively the patient is ventilating and whether rapid intubation may be necessary. The length of time the attack has persisted is not as important as determining the patient's status at present. Most patients having an acute attack will be unable to cooperate with an FEV measurement. It is important to know about the medications the patient is using but not as important as assessing the breath sounds.

Which information about a patient who is receiving cisatracurium (Nimbex) to prevent asynchronous breathing with the positive pressure ventilator requires immediate action by the nurse? a. Only continuous IV opioids have been ordered. b. The patient does not respond to verbal stimulation. c. There is no cough or gag when the patient is suctioned. d. The patient's oxygen saturation fluctuates between 90% to 93%.

ANS: A Because neuromuscular blockade is extremely anxiety provoking, it is essential that patients who are receiving neuromuscular blockade receive concurrent sedation and analgesia. Absence of response to stimuli is expected in patients receiving neuromuscular blockade. The oxygen saturation is adequate.

A child with cystic fibrosis (CF) receives aerosolized bronchodilator medication. When should this medication be administered? a. Before chest physiotherapy (CPT) b. After CPT c. Before receiving 100% oxygen d. After receiving 100% oxygen

ANS: A Bronchodilators should be given before CPT to open bronchi and make expectoration easier. Aerosolized bronchodilator medications are not helpful when used after CPT. Oxygen administration is necessary only in acute episodes with caution because of chronic carbon dioxide retention.

The nurse is caring for a patient with cor pulmonale. The nurse should monitor the patient for which expected finding? a. Peripheral edema b. Elevated temperature c. Clubbing of the fingers d. Complaints of chest pain

ANS: A Cor pulmonale causes clinical manifestations of right ventricular failure, such as peripheral edema. The other clinical manifestations may occur in the patient with other complications of chronic obstructive pulmonary disease (COPD) but are not indicators of cor pulmonale. DIF: Cognitive Level: Apply (application) REF: 586 TOP: Nursing Process: Evaluation MSC: NCLEX: Physiological Integrity

A nurse is charting that a hospitalized child has labored breathing. Which describes labored breathing? a. Dyspnea b. Tachypnea c. Hypopnea d. Orthopnea

ANS: A Dyspnea is labored breathing. Tachypnea is rapid breathing. Hypopnea is breathing that is too shallow. Orthopnea is difficulty breathing except in upright position.

A nurse is caring for a patient who is orally intubated and receiving mechanical ventilation. To decrease the risk for ventilator-associated pneumonia, which action will the nurse include in the plan of care? a. Elevate head of bed to 30 to 45 degrees. b. Suction the endotracheal tube every 2 to 4 hours. c. Limit the use of positive end-expiratory pressure. d. Give enteral feedings at no more than 10 mL/hr.

ANS: A Elevation of the head decreases the risk for aspiration. Positive end-expiratory pressure is frequently needed to improve oxygenation in patients receiving mechanical ventilation. Suctioning should be done only when the patient assessment indicates that it is necessary. Enteral feedings should provide adequate calories for the patient's high energy needs.

Which finding in a patient hospitalized with bronchiectasis is most important to report to the health care provider? a. Cough productive of bloody, purulent mucus b. Scattered rhonchi and wheezes heard bilaterally c. Respiratory rate 28 breaths/minute while ambulating in hallway d. Complaint of sharp chest pain with deep breathing

ANS: A Hemoptysis may indicate life-threatening hemorrhage and should be reported immediately to the health care provider. The other findings are frequently noted in patients with bronchiectasis and may need further assessment but are not indicators of life-threatening complications. DIF: Cognitive Level: Apply (application) REF: 607 OBJ: Special Questions: Prioritization TOP: Nursing Process: Assessment MSC: NCLEX: Safe and Effective Care Environment

While caring for a patient who has been admitted with a pulmonary embolism, the nurse notes a change in the patient's oxygen saturation (SpO2) from 94% to 88%. Which action should the nurse take next? a. Increase the oxygen flow rate. b. Suction the patient's oropharynx. c. Instruct the patient to cough and deep breathe. d. Help the patient to sit in a more upright position.

ANS: A Increasing oxygen flow rate will usually improve oxygen saturation in patients with ventilation-perfusion mismatch, as occurs with pulmonary embolism. Because the problem is with perfusion, actions that improve ventilation, such as deep breathing and coughing, sitting upright, and suctioning, are not likely to improve oxygenation.

When admitting a patient with possible respiratory failure with a high PaCO2, which assessment information should be immediately reported to the health care provider? a. The patient is somnolent. b. The patient complains of weakness. c. The patient's blood pressure is 164/98. d. The patient's oxygen saturation is 90%.

ANS: A Increasing somnolence will decrease the patient's respiratory rate and further increase the PaCO2 and respiratory failure. Rapid action is needed to prevent respiratory arrest. An SpO2 of 90%, weakness, and elevated blood pressure all require ongoing monitoring but are not indicators of possible impending respiratory arrest.

A nurse is caring for a patient with acute respiratory distress syndrome (ARDS) who is receiving mechanical ventilation using synchronized intermittent mandatory ventilation (SIMV). The settings include fraction of inspired oxygen (FIO2) 80%, tidal volume 450, rate 16/minute, and positive end-expiratory pressure (PEEP) 5 cm. Which assessment finding is most important for the nurse to report to the health care provider? a. Oxygen saturation 99% b. Respiratory rate 22 breaths/minute c. Crackles audible at lung bases d. Heart rate 106 beats/minute

ANS: A The FIO2 of 80% increases the risk for oxygen toxicity. Because the patient's O2 saturation is 99%, a decrease in FIO2 is indicated to avoid toxicity. The other patient data would be typical for a patient with ARDS and would not need to be urgently reported to the health care provider.

The nurse is caring for a 33-year-old patient who arrived in the emergency department with acute respiratory distress. Which assessment finding by the nurse requires the most rapid action? a. The patient's PaO2 is 45 mm Hg. b. The patient's PaCO2 is 33 mm Hg. c. The patient's respirations are shallow. d. The patient's respiratory rate is 32 breaths/minute.

ANS: A The PaO2 indicates severe hypoxemia and respiratory failure. Rapid action is needed to prevent further deterioration of the patient. Although the shallow breathing, rapid respiratory rate, and low PaCO2 also need to be addressed, the most urgent problem is the patient's poor oxygenation.

The nurse in the emergency department receives arterial blood gas results for four recently admitted patients with obstructive pulmonary disease. Which patient will require the most rapid action by the nurse? a. 22-year-old with ABG results: pH 7.28, PaCO2 60 mm Hg, and PaO2 58 mm Hg b. 34-year-old with ABG results: pH 7.48, PaCO2 30 mm Hg, and PaO2 65 mm Hg c. 45-year-old with ABG results: pH 7.34, PaCO2 33 mm Hg, and PaO2 80 mm Hg d. 65-year-old with ABG results: pH 7.31, PaCO2 58 mm Hg, and PaO2 64 mm Hg

ANS: A The pH, PaCO2, and PaO2 indicate that the patient has severe uncompensated respiratory acidosis and hypoxemia. Rapid action will be required to prevent increasing hypoxemia and correct the acidosis. The other patients also should be assessed as quickly as possible but do not require interventions as quickly as the 22-year-old. DIF: Cognitive Level: Analyze (analysis) REF: 566 OBJ: Special Questions: Prioritization; Multiple Patients TOP: Nursing Process: Assessment MSC: NCLEX: Safe and Effective Care Environment

A nurse is caring for an obese patient with right lower lobe pneumonia. Which position will be best to improve gas exchange? a. On the left side b. On the right side c. In the tripod position d. In the high-Fowler's position

ANS: A The patient should be positioned with the "good" lung in the dependent position to improve the match between ventilation and perfusion. The obese patient's abdomen will limit respiratory excursion when sitting in the high-Fowler's or tripod positions.

A patient who is experiencing an asthma attack develops bradycardia and a decrease in wheezing. Which action should the nurse take first? a. Notify the health care provider. b. Document changes in respiratory status. c. Encourage the patient to cough and deep breathe. d. Administer IV methylprednisolone (Solu-Medrol).

ANS: A The patient's assessment indicates impending respiratory failure, and the nurse should prepare to assist with intubation and mechanical ventilation after notifying the health care provider. IV corticosteroids require several hours before having any effect on respiratory status. The patient will not be able to cough or deep breathe effectively. Documentation is not a priority at this time.

The nurse reviews the electronic medical record for a patient scheduled for a total hip replacement. Which assessment data shown in the accompanying figure increase the patient's risk for respiratory complications after surgery? a. Albumin level and recent weight loss b. Mild confusion and recent weight loss c. Age and recent arthroscopic procedure. d. Anemia and recent arthroscopic procedure

ANS: A The patient's recent weight loss and low protein stores indicate possible muscle weakness, which make it more difficult for an older patient to recover from the effects of general anesthesia and immobility associated with the hip surgery. The other information will also be noted by the nurse but does not place the patient at higher risk for respiratory failure.

A patient with acute respiratory distress syndrome (ARDS) is placed in the prone position. When prone positioning is used, which information obtained by the nurse indicates that the positioning is effective? a. The patient's PaO2 is 89 mm Hg, and the SaO2 is 91%. b. Endotracheal suctioning results in clear mucous return. c. Sputum and blood cultures show no growth after 48 hours. d. The skin on the patient's back is intact and without redness

ANS: A The purpose of prone positioning is to improve the patient's oxygenation as indicated by the PaO2 and SaO2. The other information will be collected but does not indicate whether prone positioning has been effective.

Which nursing action for a patient with chronic obstructive pulmonary disease (COPD) could the nurse delegate to experienced unlicensed assistive personnel (UAP)? a. Obtain oxygen saturation using pulse oximetry. b. Monitor for increased oxygen need with exercise. c. Teach the patient about safe use of oxygen at home. d. Adjust oxygen to keep saturation in prescribed parameters.

ANS: A UAP can obtain oxygen saturation (after being trained and evaluated in the skill). The other actions require more education and a scope of practice that licensed practical/vocational nurses (LPN/LVNs) or registered nurses (RNs) would have. DIF: Cognitive Level: Apply (application) REF: 592 OBJ: Special Questions: Delegation TOP: Nursing Process: Planning MSC: NCLEX: Safe and Effective Care Environment

A young adult patient who denies any history of smoking is seen in the clinic with a new diagnosis of chronic obstructive pulmonary disease (COPD). It is most appropriate for the nurse to teach the patient about a. α1-antitrypsin testing. b. use of the nicotine patch. c. continuous pulse oximetry. d. effects of leukotriene modifiers.

ANS: A When COPD occurs in young patients, especially without a smoking history, a genetic deficiency in α1-antitrypsin should be suspected. Because the patient does not smoke, a nicotine patch would not be ordered. There is no indication that the patient requires continuous pulse oximetry. Leukotriene modifiers would be used in patients with asthma, not with COPD. DIF: Cognitive Level: Apply (application) REF: 582-583 TOP: Nursing Process: Planning MSC: NCLEX: Physiological Integrity

Which information should the nurse teach families about reducing exposure to pollens and dust (Select all that apply)? a. Replace wall-to-wall carpeting with wood and tile floors. b. Use an air conditioner. c. Put dust-proof covers on pillows and mattresses. d. Keep humidity in the house above 60%. e. Keep pets outside.

ANS: A, B, C Carpets retain dust. To reduce exposure to dust, carpeting should be replaced with wood, tile, slate, or vinyl. These floors can be cleaned easily. For anyone with pollen allergies, it is best to keep the windows closed and to run the air conditioner. Covering mattresses and pillows with dust-proof covers will reduce exposure to dust. A humidity level above 60% promotes dust mites. It is recommended that household humidity be kept between 40% and 50% to reduce dust mites inside the house. Keeping pets outside will help to decrease exposure to dander, but will not affect exposure to pollen and dust.

An infant has developed staphylococcal pneumonia. Nursing care of the child with pneumonia includes which of the following? (Select all that apply). a. Cluster care to conserve energy b. Round-the-clock administration of antitussive agents c. Strict intake and output to avoid congestive heart failure d. Administration of antibiotics e. Placement in a mist tent

ANS: A, D Antibiotics are indicated for a bacterial pneumonia. Often the child will have decreased pulmonary reserve, and the clustering of care is essential. Round-the-clock antitussive agents and strict intake and output are not included in the care of the child with pneumonia. Mist tents are no longer utilized for pediatric respiratory care. PTS: 1 DIF: Cognitive Level: Comprehension REF: 1214 OBJ: Nursing Process: Implementation MSC:

The nurse is caring for a child with carbon monoxide (CO) poisoning associated with smoke inhalation. What is essential in this childs care? a. Monitor pulse oximetry. b. Monitor arterial blood gases. c. Administer oxygen if respiratory distress develops. d. Administer oxygen if childs lips become bright, cherry red.

ANS: B Arterial blood gases and COHb levels are the best way to monitor CO poisoning. PaO2 monitored with pulse oximetry may be normal in the case of CO poisoning. Oxygen at 100% should be given as quickly as possible, not only if respiratory distress or other symptoms develop. PTS: 1 DIF: Cognitive Level: Comprehension REF: 1222 OBJ: Nursing Process: Assessment MSC:

Which statement best describes why children have fewer respiratory tract infections as they grow older? a. The amount of lymphoid tissue decreases. b. Repeated exposure to organisms causes increased immunity. c. Viral organisms are less prevalent in the population. d. Secondary infections rarely occur after viral illnesses.

ANS: B Children have increased immunity after exposure to a virus. The amount of lymphoid tissue increases as children grow older. Viral organisms are not less prevalent, but older children have the ability to resist invading organisms. Secondary infections after viral illnesses include Mycoplasma pneumoniae and groups A and B streptococcal infections. PTS: 1 DIF: Cognitive Level: Comprehension REF: 1195 OBJ: Nursing Process: Assessment MSC:

Cardiopulmonary resuscitation is begun on a toddler. Which pulse is usually palpated because it is the most central and accessible? a. Radial b. Carotid c. Femoral d. Brachial

ANS: B In a toddler, the carotid pulse is palpated. The radial pulse is not considered a central pulse. The femoral pulse is not the most central and accessible. The brachial pulse is felt in infants younger than 1 year.

The nurse is caring for a child with acute respiratory distress syndrome (ARDS) associated with sepsis. Nursing actions should include: a. Force fluids. b. Monitor pulse oximetry. c. Institute seizure precautions. d. Encourage a high-protein diet.

ANS: B Monitoring cardiopulmonary status is an important evaluation tool in the care of the child with ARDS. Maintenance of vascular volume and hydration is important and should be done parenterally. Seizures are not a side effect of ARDS. Adequate nutrition is necessary, but a high-protein diet is not helpful. PTS: 1 DIF: Cognitive Level: Comprehension REF: 1221 OBJ: Nursing Process: Implementation MSC:

The parent of a toddler calls the nurse, asking about croup. What is a distinguishing manifestation of spasmodic croup? a. Wheezing is heard audibly. b. It has a harsh, barky cough. c. It is bacterial in nature. d. The child has a high fever.

ANS: B Spasmodic croup is viral in origin, is usually preceded by several days of symptoms of upper respiratory tract infection, and often begins at night. It is marked by a harsh, metallic, barky cough; sore throat; inspiratory stridor; and hoarseness. Wheezing is not a distinguishing manifestation of croup. It can accompany conditions such as asthma or bronchiolitis. A high fever is not usually present.

The nurse encourages the mother of a toddler with acute laryngotracheobronchitis to stay at the bedside as much as possible. The nurses rationale for this action is primarily that: a. Mothers of hospitalized toddlers often experience guilt. b. The mothers presence will reduce anxiety and ease the childs respiratory efforts. c. Separation from the mother is a major developmental threat at this age. d. The mother can provide constant observations of the childs respiratory efforts.

ANS: B The familys presence will decrease the childs distress. The mother may experience guilt, but this is not the best answer. Although separation from the mother is a developmental threat for toddlers, the main reason to keep parents at the childs bedside is to ease anxiety and therefore respiratory effort. The child should have constant cardiorespiratory monitoring and noninvasive oxygen saturation monitoring, but the parent should not play this role in the hospital. PTS: 1 DIF: Cognitive Level: Application REF: 1210 OBJ: Nursing Process: Implementation

A child with cystic fibrosis is receiving recombinant human deoxyribonuclease (rhDNase). This drug: a. May cause mucus to thicken. b. May cause voice alterations. c. Is given subcutaneously. d. Is not indicated for children younger than 12 years.

ANS: B Two of the only adverse effects of rhDNase are voice alterations and laryngitis. rhDNase decreases viscosity of mucus, is given in an aerosolized form, and is safe for children younger than 12 years of age. PTS: 1 DIF: Cognitive Level: Comprehension REF: 1237 OBJ: Nursing Process: Implementation MSC:

Asthma in infants is usually triggered by: a. Medications. b. A viral infection. c. Exposure to cold air. d. Allergy to dust or dust mites.

ANS: B Viral illnesses cause inflammation that causes increased airway reactivity in asthma. Medications such as aspirin, nonsteroidal antiinflammatory drugs, and antibiotics may aggravate asthma, but not frequently in infants. Exposure to cold air may exacerbate already existing asthma. Allergy is associated with asthma, but 20% to 40% of children with asthma have no evidence of allergic disease.

Which consideration is the most important in managing tuberculosis (TB) in children? a. Skin testing annually b. Pharmacotherapy c. Adequate nutrition d. Adequate hydration

ANS: B Drug therapy for TB includes isoniazid, rifampin, and pyrazinamide daily for 2 months and 2 or 3 times a week for the remaining 4 months. Although skin testing and adequate nutrition and hydration are important, pharmacotherapy is the most important intervention for TB. PTS: 1 DIF: Cognitive Level: Application REF: 1218 OBJ: Nursing Process: Implementation MSC:

The nurse receives a change-of-shift report on the following patients with chronic obstructive pulmonary disease (COPD). Which patient should the nurse assess first? a. A patient with loud expiratory wheezes b. A patient with a respiratory rate of 38/minute c. A patient who has a cough productive of thick, green mucus d. A patient with jugular venous distention and peripheral edema

ANS: B A respiratory rate of 38/minute indicates severe respiratory distress, and the patient needs immediate assessment and intervention to prevent possible respiratory arrest. The other patients also need assessment as soon as possible, but they do not need to be assessed as urgently as the tachypneic patient.

A patient with chronic obstructive pulmonary disease (COPD) has rhonchi throughout the lung fields and a chronic, nonproductive cough. Which nursing intervention will be most effective? a. Change the oxygen flow rate to the highest prescribed rate. b. Teach the patient to use the Flutter airway clearance device. c. Reinforce the ongoing use of pursed lip breathing techniques. d. Teach the patient about consistent use of inhaled corticosteroids.

ANS: B Airway clearance devices assist with moving mucus into larger airways where it can more easily be expectorated. The other actions may be appropriate for some patients with COPD, but they are not indicated for this patient's problem of thick mucus secretions. DIF: Cognitive Level: Apply (application) REF: 595 TOP: Nursing Process: Implementation MSC: NCLEX: Physiological Integrity

A patient admitted with acute respiratory failure has a nursing diagnosis of ineffective airway clearance related to thick, secretions. Which action is a priority for the nurse to include in the plan of care? a. Encourage use of the incentive spirometer. b. Offer the patient fluids at frequent intervals. c. Teach the patient the importance of ambulation. d. Titrate oxygen level to keep O2 saturation >93%.

ANS: B Because the reason for the poor airway clearance is the thick secretions, the best action will be to encourage the patient to improve oral fluid intake. Patients should be instructed to use the incentive spirometer on a regular basis (e.g., every hour) in order to facilitate the clearance of the secretions. The other actions may also be helpful in improving the patient's gas exchange, but they do not address the thick secretions that are causing the poor airway clearance.

Which statement by the nurse when explaining the purpose of positive end-expiratory pressure (PEEP) to the family members of a patient with ARDS is accurate? a. "PEEP will push more air into the lungs during inhalation." b. "PEEP prevents the lung air sacs from collapsing during exhalation." c. "PEEP will prevent lung damage while the patient is on the ventilator." d. "PEEP allows the breathing machine to deliver 100% oxygen to the lungs."

ANS: B By preventing alveolar collapse during expiration, PEEP improves gas exchange and oxygenation. PEEP will not prevent lung damage (e.g., fibrotic changes that occur with ARDS), push more air into the lungs, or change the fraction of inspired oxygen (FIO2) delivered to the patient.

Which type of croup is always considered a medical emergency? a. Laryngitis b. Epiglottitis c. Spasmodic croup d. Laryngotracheobronchitis (LTB)

ANS: B Epiglottitis is always a medical emergency needing antibiotics and airway support for treatment. Laryngitis is a common viral illness in older children and adolescents, with hoarseness and upper respiratory infection symptoms. Spasmodic croup is treated with humidity. LTB may progress to a medical emergency in some children.

Which assessment finding in a patient who has received omalizumab (Xolair) is most important to report immediately to the health care provider? a. Pain at injection site b. Flushing and dizziness c. Peak flow reading 75% of normal d. Respiratory rate 22 breaths/minute

ANS: B Flushing and dizziness may indicate that the patient is experiencing an anaphylactic reaction, and immediate intervention is needed. The other information should also be reported, but do not indicate possibly life-threatening complications of omalizumab therapy. DIF: Cognitive Level: Apply (application) REF: 572 OBJ: Special Questions: Prioritization TOP: Nursing Process: Assessment MSC: NCLEX: Physiological Integrity

Which finding by the nurse for a patient with a nursing diagnosis of impaired gas exchange will be most useful in evaluating the effectiveness of treatment? a. Even, unlabored respirations b. Pulse oximetry reading of 92% c. Respiratory rate of 18 breaths/minute d. Absence of wheezes, rhonchi, or crackles

ANS: B For the nursing diagnosis of impaired gas exchange, the best data for evaluation are arterial blood gases (ABGs) or pulse oximetry. The other data may indicate either improvement or impending respiratory failure caused by fatigue. DIF: Cognitive Level: Apply (application) REF: 598 TOP: Nursing Process: Evaluation MSC: NCLEX: Physiological Integrity

A patient with acute respiratory distress syndrome (ARDS) and acute kidney injury has the following medications ordered. Which medication should the nurse discuss with the health care provider before giving? a. Pantoprazole (Protonix) 40 mg IV b. Gentamicin (Garamycin) 60 mg IV c. Sucralfate (Carafate) 1 g per nasogastric tube d. Methylprednisolone (Solu-Medrol) 60 mg IV

ANS: B Gentamicin, which is one of the aminoglycoside antibiotics, is potentially nephrotoxic, and the nurse should clarify the drug and dosage with the health care provider before administration. The other medications are appropriate for the patient with ARDS.

The nurse provides dietary teaching for a patient with chronic obstructive pulmonary disease (COPD) who has a low body mass index (BMI). Which patient statement indicates that the teaching has been effective? a. "I will drink lots of fluids with my meals." b. "I can have ice cream as a snack every day." c. "I will exercise for 15 minutes before meals." d. "I will decrease my intake of meat and poultry."

ANS: B High-calorie foods like ice cream are an appropriate snack for patients with COPD. Fluid intake of 3 L/day is recommended, but fluids should be taken between meals rather than with meals to improve oral intake of solid foods. The patient should avoid exercise for an hour before meals to prevent fatigue while eating. Meat and dairy products are high in protein and are good choices for the patient with COPD. DIF: Cognitive Level: Apply (application) REF: 595-596 TOP: Nursing Process: Evaluation MSC: NCLEX: Physiological Integrity

The nurse is assessing a child with acute epiglottitis. Examining the childs throat by using a tongue depressor might precipitate which symptom or condition? a. Inspiratory stridor b. Complete obstruction c. Sore throat d. Respiratory tract infection

ANS: B If a child has acute epiglottitis, examination of the throat may cause complete obstruction and should be performed only when immediate intubation can take place. Stridor is aggravated when a child with epiglottitis is supine. Sore throat and pain on swallowing are early signs of epiglottitis. Epiglottitis is caused by Haemophilus influenzae in the respiratory tract.

A patient is receiving 35% oxygen via a Venturi mask. To ensure the correct amount of oxygen delivery, which action by the nurse is most important? a. Teach the patient to keep mask on at all times. b. Keep the air entrainment ports clean and unobstructed. c. Give a high enough flow rate to keep the bag from collapsing. d. Drain moisture condensation from the oxygen tubing every hour.

ANS: B The air entrainment ports regulate the oxygen percentage delivered to the patient, so they must be unobstructed. A high oxygen flow rate is needed when giving oxygen by partial rebreather or non-rebreather masks. Draining oxygen tubing is necessary when caring for a patient receiving mechanical ventilation. The mask is uncomfortable and can be removed when the patient eats. DIF: Cognitive Level: Apply (application) REF: 591 TOP: Nursing Process: Implementation MSC: NCLEX: Physiological Integrity

The nurse is admitting a patient diagnosed with an acute exacerbation of chronic obstructive pulmonary disease (COPD).What is the best way for the nurse to determine the appropriate oxygen flow rate? a. Minimize oxygen use to avoid oxygen dependency. b. Maintain the pulse oximetry level at 90% or greater. c. Administer oxygen according to the patient's level of dyspnea. d. Avoid administration of oxygen at a rate of more than 2 L/minute.

ANS: B The best way to determine the appropriate oxygen flow rate is by monitoring the patient's oxygenation either by arterial blood gases (ABGs) or pulse oximetry. An oxygen saturation of 90% indicates adequate blood oxygen level without the danger of suppressing the respiratory drive. For patients with an exacerbation of COPD, an oxygen flow rate of 2 L/min may not be adequate. Because oxygen use improves survival rate in patients with COPD, there is no concern about oxygen dependency. The patient's perceived dyspnea level may be affected by other factors (such as anxiety) besides blood oxygen level. DIF: Cognitive Level: Apply (application) REF: 589 TOP: Nursing Process: Implementation MSC: NCLEX: Physiological Integrity

A patient with cystic fibrosis (CF) has blood glucose levels that are consistently between 180 to 250 mg/dL. Which nursing action will the nurse plan to implement? a. Discuss the role of diet in blood glucose control. b. Teach the patient about administration of insulin. c. Give oral hypoglycemic medications before meals. d. Evaluate the patient's home use of pancreatic enzymes.

ANS: B The glucose levels indicate that the patient has developed CF-related diabetes, and insulin therapy is required. Because the etiology of diabetes in CF is inadequate insulin production, oral hypoglycemic agents are not effective. Patients with CF need a high-calorie diet. Inappropriate use of pancreatic enzymes would not be a cause of hyperglycemia in a patient with CF. DIF: Cognitive Level: Apply (application) REF: 603 TOP: Nursing Process: Planning MSC: NCLEX: Physiological Integrity

The emergency department nurse is evaluating the effectiveness of therapy for a patient who has received treatment during an asthma attack. Which assessment finding is the best indicator that the therapy has been effective? a. No wheezes are audible. b. Oxygen saturation is >90%. c. Accessory muscle use has decreased. d. Respiratory rate is 16 breaths/minute.

ANS: B The goal for treatment of an asthma attack is to keep the oxygen saturation >90%. The other patient data may occur when the patient is too fatigued to continue with the increased work of breathing required in an asthma attack. DIF: Cognitive Level: Apply (application) REF: 569 TOP: Nursing Process: Evaluation MSC: NCLEX: Physiological Integrity

The nurse is caring for a patient who is intubated and receiving positive pressure ventilation to treat acute respiratory distress syndrome (ARDS). Which finding is most important to report to the health care provider? a. Blood urea nitrogen (BUN) level 32 mg/dL b. Red-brown drainage from orogastric tube c. Scattered coarse crackles heard throughout lungs d. Arterial blood gases: pH 7.31, PaCO2 50, PaO2 68

ANS: B The nasogastric drainage indicates possible gastrointestinal bleeding and/or stress ulcer, and should be reported. The pH and PaCO2 are slightly abnormal, but current guidelines advocating for permissive hypercapnia indicate that these would not indicate an immediate need for a change in therapy. The BUN is slightly elevated but does not indicate an immediate need for action. Adventitious breath sounds are commonly heard in patients with ARDS.

A young adult female patient with cystic fibrosis (CF) tells the nurse that she is not sure about getting married and having children some day. Which initial response by the nurse is best? a. "Are you aware of the normal lifespan for patients with CF?" b. "Do you need any information to help you with that decision?" c. "Many women with CF do not have difficulty conceiving children." d. "You will need to have genetic counseling before making a decision."

ANS: B The nurse's initial response should be to assess the patient's knowledge level and need for information. Although the lifespan for patients with CF is likely to be shorter than normal, it would not be appropriate for the nurse to address this as the initial response to the patient's comments. The other responses have accurate information, but the nurse should first assess the patient's understanding about the issues surrounding pregnancy. DIF: Cognitive Level: Apply (application) REF: 605-606 TOP: Nursing Process: Implementation MSC: NCLEX: Health Promotion and Maintenance

The nurse teaches a patient about pursed lip breathing. Which action by the patient would indicate to the nurse that further teaching is needed? a. The patient inhales slowly through the nose. b. The patient puffs up the cheeks while exhaling. c. The patient practices by blowing through a straw. d. The patient's ratio of inhalation to exhalation is 1:3.

ANS: B The patient should relax the facial muscles without puffing the cheeks while doing pursed lip breathing. The other actions by the patient indicate a good understanding of pursed lip breathing. DIF: Cognitive Level: Apply (application) REF: 579 TOP: Nursing Process: Evaluation MSC: NCLEX: Physiological Integrity

The oxygen saturation (SpO2) for a patient with left lower lobe pneumonia is 90%. The patient has rhonchi, a weak cough effort, and complains of fatigue. Which action is a priority for the nurse to take? a. Position the patient on the left side. b. Assist the patient with staged coughing. c. Place a humidifier in the patient's room. d. Schedule a 2-hour rest period for the patient.

ANS: B The patient's assessment indicates that assisted coughing is needed to help remove secretions, which will improve oxygenation. A 2-hour rest period at this time may allow the oxygen saturation to drop further. Humidification will not be helpful unless the secretions can be mobilized. Positioning on the left side may cause a further decrease in oxygen saturation because perfusion will be directed more toward the more poorly ventilated lung.

A patient with respiratory failure has a respiratory rate of 6 breaths/minute and an oxygen saturation (SpO2) of 88%. The patient is increasingly lethargic. Which intervention will the nurse anticipate? a. Administration of 100% oxygen by non-rebreather mask b. Endotracheal intubation and positive pressure ventilation c. Insertion of a mini-tracheostomy with frequent suctioning d. Initiation of continuous positive pressure ventilation (CPAP)

ANS: B The patient's lethargy, low respiratory rate, and SpO2 indicate the need for mechanical ventilation with ventilator-controlled respiratory rate. Administration of high flow oxygen will not be helpful because the patient's respiratory rate is so low. Insertion of a mini-tracheostomy will facilitate removal of secretions, but it will not improve the patient's respiratory rate or oxygenation. CPAP requires that the patient initiate an adequate respiratory rate to allow adequate gas exchange.

The clinic nurse makes a follow-up telephone call to a patient with asthma. The patient reports having a baseline peak flow reading of 600 L/minute and the current peak flow is 420 L/minute. Which action should the nurse take first? a. Tell the patient to go to the hospital emergency department. b. Instruct the patient to use the prescribed albuterol (Proventil). c. Ask about recent exposure to any new allergens or asthma triggers. d. Question the patient about use of the prescribed inhaled corticosteroids.

ANS: B The patient's peak flow is 70% of normal, indicating a need for immediate use of short-acting β2-adrenergic SABA medications. Assessing for correct use of medications or exposure to allergens also is appropriate, but would not address the current decrease in peak flow. Because the patient is currently in the yellow zone, hospitalization is not needed. DIF: Cognitive Level: Analyze (analysis) REF: 580 OBJ: Special Questions: Prioritization TOP: Nursing Process: Implementation MSC: NCLEX: Physiological Integrity

A nurse is caring for a patient with ARDS who is being treated with mechanical ventilation and high levels of positive end-expiratory pressure (PEEP). Which assessment finding by the nurse indicates that the PEEP may need to be reduced? a. The patient's PaO2 is 50 mm Hg and the SaO2 is 88%. b. The patient has subcutaneous emphysema on the upper thorax. c. The patient has bronchial breath sounds in both the lung fields. d. The patient has a first-degree atrioventricular heart block with a rate of 58.

ANS: B The subcutaneous emphysema indicates barotrauma caused by positive pressure ventilation and PEEP. Bradycardia, hypoxemia, and bronchial breath sounds are all concerns and will need to be addressed, but they are not specific indications that PEEP should be reduced.

b-Adrenergic agonists and methylxanthines are often prescribed for a child with an asthma attack. What is their action? a. Liquefy secretions b. Dilate the bronchioles c. Reduce inflammation of the lungs d. Reduce infection

ANS: B These medications work to dilate the bronchioles in acute exacerbations. These medications do not liquefy secretions or reduce infection. Corticosteroids and mast cell stabilizers reduce inflammation in the lungs.

Which instruction should the nurse include in an exercise teaching plan for a patient with chronic obstructive pulmonary disease (COPD)? a. "Stop exercising if you start to feel short of breath." b. "Use the bronchodilator before you start to exercise." c. "Breathe in and out through the mouth while you exercise." d. "Upper body exercise should be avoided to prevent dyspnea."

ANS: B Use of a bronchodilator before exercise improves airflow for some patients and is recommended. Shortness of breath is normal with exercise and not a reason to stop. Patients should be taught to breathe in through the nose and out through the mouth (using a pursed lip technique). Upper-body exercise can improve the mechanics of breathing in patients with COPD. DIF: Cognitive Level: Apply (application) REF: 572 TOP: Nursing Process: Implementation MSC: NCLEX: Physiological Integrity

It is generally recommended that a child with acute streptococcal pharyngitis can return to school: a. When the sore throat is better. b. If no complications develop. c. After taking antibiotics for 24 hours. d. After taking antibiotics for 3 days.

ANS: C After children have taken antibiotics for 24 hours, even if the sore throat persists, they are no longer contagious to other children. Complications may take days to weeks to develop. PTS: 1 DIF: Cognitive Level: Comprehension REF: 1201 OBJ: Nursing Process: Implementation MSC:

A child is diagnosed with influenza, probably type A disease. Management includes: a. Clear liquid diet for hydration. b. Aspirin to control fever. c. Amantadine hydrochloride to reduce symptoms. d. Antibiotics to prevent bacterial infection.

ANS: C Amantadine hydrochloride may reduce symptoms related to influenza type A if administered within 24 to 48 hours of onset. It is ineffective against type B or C. A clear liquid diet is not necessary for influenza, but maintaining hydration is important. Aspirin is not recommended in children because of increased risk of Reyes syndrome. Acetaminophen or ibuprofen is a better choice. Preventive antibiotics are not indicated for influenza unless there is evidence of a secondary bacterial infection. PTS: 1 DIF: Cognitive Level: Application REF: 1204 OBJ: Nursing Process: Implementation MSC:

A parent whose two school-age children have asthma asks the nurse in what sports, if any, they can participate. The nurse should recommend: a. Soccer. b. Running. c. Swimming. d. Basketball.

ANS: C Swimming is well tolerated in children with asthma because they are breathing air fully saturated with moisture and because of the type of breathing required in swimming. Exercise-induced bronchospasm is more common in sports that involve endurance, such as soccer, running, and basketball. Prophylaxis with medications may be necessary.

Which intervention for treating croup at home should be taught to parents? a. Have a decongestant available to give the child when an attack occurs. b. Have the child sleep in a dry room. c. Take the child outside. d. Give the child an antibiotic at bedtime.

ANS: C Taking the child into the cool, humid, night air may relieve mucosal swelling and improve symptoms. Decongestants are inappropriate for croup, which affects the middle airway level. A dry environment may contribute to symptoms. Croup is caused by a virus. Antibiotic treatment is not indicated.

It is now recommended that children with asthma who are taking long-term inhaled steroids should be assessed frequently because they may develop: a. Cough. b. Osteoporosis. c. Slowed growth. d. Cushings syndrome.

ANS: C The growth of children on long-term inhaled steroids should be assessed frequently to assess for systemic effects of these drugs. Cough is prevented by inhaled steroids. No evidence exists that inhaled steroids cause osteoporosis. Cushings syndrome is caused by long-term systemic steroids. PTS: 1 DIF: Cognitive Level: Comprehension REF: 1228 OBJ: Nursing Process: Diagnosis MSC:

A patient with chronic obstructive pulmonary disease (COPD) arrives in the emergency department complaining of shortness of breath and dyspnea on minimal exertion. Which assessment finding by the nurse is most important to report to the health care provider? a. The patient has bibasilar lung crackles. b. The patient is sitting in the tripod position. c. The patient's respirations have decreased from 30 to 10 breaths/minute. d. The patient's pulse oximetry indicates an O2 saturation of 91%.

ANS: C A decrease in respiratory rate in a patient with respiratory distress suggests the onset of fatigue and a high risk for respiratory arrest. Therefore immediate action such as positive pressure ventilation is needed. Patients who are experiencing respiratory distress frequently sit in the tripod position because it decreases the work of breathing. Crackles in the lung bases may be the baseline for a patient with COPD. An oxygen saturation of 91% is common in patients with COPD and will provide adequate gas exchange and tissue oxygenation.

The nurse interviews a patient with a new diagnosis of chronic obstructive pulmonary disease (COPD). Which information is most helpful in confirming a diagnosis of chronic bronchitis? a. The patient tells the nurse about a family history of bronchitis. b. The patient's history indicates a 30 pack-year cigarette history. c. The patient complains about a productive cough every winter for 3 months. d. The patient denies having any respiratory problems until the last 12 months.

ANS: C A diagnosis of chronic bronchitis is based on a history of having a productive cough for 3 months for at least 2 consecutive years. There is no family tendency for chronic bronchitis. Although smoking is the major risk factor for chronic bronchitis, a smoking history does not confirm the diagnosis. DIF: Cognitive Level: Apply (application) REF: 579 TOP: Nursing Process: Assessment MSC: NCLEX: Physiological Integrity

During change-of-shift report on a medical unit, the nurse learns that a patient with aspiration pneumonia who was admitted with respiratory distress has become increasingly agitated. Which action should the nurse take first? a. Give the prescribed PRN sedative drug. b. Offer reassurance and reorient the patient. c. Use pulse oximetry to check the oxygen saturation. d. Notify the health care provider about the patient's status.

ANS: C Agitation may be an early indicator of hypoxemia. The other actions may also be appropriate, depending on the findings about oxygen saturation.

To evaluate the effectiveness of ordered interventions for a patient with ventilatory failure, which diagnostic test will be most useful to the nurse? a. Chest x-ray b. Oxygen saturation c. Arterial blood gas analysis d. Central venous pressure monitoring

ANS: C Arterial blood gas (ABG) analysis is most useful in this setting because ventilatory failure causes problems with CO2 retention, and ABGs provide information about the PaCO2 and pH. The other tests may also be done to help in assessing oxygenation or determining the cause of the patient's ventilatory failure.

A patient is scheduled for pulmonary function testing. Which action should the nurse take to prepare the patient for this procedure? a. Give the rescue medication immediately before testing. b. Administer oral corticosteroids 2 hours before the procedure. c. Withhold bronchodilators for 6 to 12 hours before the examination. d. Ensure that the patient has been NPO for several hours before the test.

ANS: C Bronchodilators are held before pulmonary function testing (PFT) so that a baseline assessment of airway function can be determined. Testing is repeated after bronchodilator use to determine whether the decrease in lung function is reversible. There is no need for the patient to be NPO. Oral corticosteroids should be held before PFTs. Rescue medications (which are bronchodilators) would not be given until after the baseline pulmonary function was assessed.

The nurse is caring for a patient with chronic obstructive pulmonary disease (COPD). Which information obtained from the patient would prompt the nurse to consult with the health care provider before administering the prescribed theophylline? a. The patient reports a recent 15-pound weight gain. b. The patient denies any shortness of breath at present. c. The patient takes cimetidine (Tagamet) 150 mg daily. d. The patient complains about coughing up green mucus.

ANS: C Cimetidine interferes with the metabolism of theophylline, and concomitant administration may lead rapidly to theophylline toxicity. The other patient information would not affect whether the theophylline should be administered or not. DIF: Cognitive Level: Apply (application) REF: 571 | 572 TOP: Nursing Process: Assessment MSC: NCLEX: Physiological Integrity

A patient with chronic obstructive pulmonary disease (COPD) has a nursing diagnosis of imbalanced nutrition: less than body requirements. Which intervention would be most appropriate for the nurse to include in the plan of care? a. Encourage increased intake of whole grains. b. Increase the patient's intake of fruits and fruit juices. c. Offer high-calorie snacks between meals and at bedtime. d. Assist the patient in choosing foods with high vegetable and mineral content.

ANS: C Eating small amounts more frequently (as occurs with snacking) will increase caloric intake by decreasing the fatigue and feelings of fullness associated with large meals. Patients with COPD should rest before meals. Foods that have a lot of texture like whole grains may take more energy to eat and get absorbed and lead to decreased intake. Although fruits, juices, and vegetables are not contraindicated, foods high in protein are a better choice. DIF: Cognitive Level: Apply (application) REF: 596 TOP: Nursing Process: Planning MSC: NCLEX: Physiological Integrity

A patient newly diagnosed with asthma is being discharged. The nurse anticipates including which topic in the discharge teaching? a. Use of long-acting β-adrenergic medications b. Side effects of sustained-release theophylline c. Self-administration of inhaled corticosteroids d. Complications associated with oxygen therapy

ANS: C Inhaled corticosteroids are more effective in improving asthma than any other drug and are indicated for all patients with persistent asthma. The other therapies would not typically be first-line treatments for newly diagnosed asthma. DIF: Cognitive Level: Apply (application) REF: 569 TOP: Nursing Process: Implementation MSC: NCLEX: Physiological Integrity

A patient with chronic obstructive pulmonary disease (COPD) has poor gas exchange. Which action by the nurse would be most appropriate? a. Have the patient rest in bed with the head elevated to 15 to 20 degrees. b. Ask the patient to rest in bed in a high-Fowler's position with the knees flexed. c. Encourage the patient to sit up at the bedside in a chair and lean slightly forward. d. Place the patient in the Trendelenburg position with several pillows behind the head.

ANS: C Patients with COPD improve the mechanics of breathing by sitting up in the "tripod" position. Resting in bed with the head elevated in a semi-Fowler's position would be an alternative position if the patient was confined to bed, but sitting in a chair allows better ventilation. The Trendelenburg position or sitting upright in bed with the knees flexed would decrease the patient's ability to ventilate well. DIF: Cognitive Level: Apply (application) REF: 599 TOP: Nursing Process: Implementation MSC: NCLEX: Physiological Integrity

A patient develops increasing dyspnea and hypoxemia 2 days after heart surgery. To determine whether the patient has acute respiratory distress syndrome (ARDS) or pulmonary edema caused by heart failure, the nurse will plan to assist with a. obtaining a ventilation-perfusion scan. b. drawing blood for arterial blood gases. c. insertion of a pulmonary artery catheter. d. positioning the patient for a chest x-ray.

ANS: C Pulmonary artery wedge pressures are normal in the patient with ARDS because the fluid in the alveoli is caused by increased permeability of the alveolar-capillary membrane rather than by the backup of fluid from the lungs (as occurs in cardiogenic pulmonary edema). The other tests will not help in differentiating cardiogenic from noncardiogenic pulmonary edema.

The nurse teaches a patient with chronic bronchitis about a new prescription for Advair Diskus (combined fluticasone and salmeterol). Which action by the patient would indicate to the nurse that teaching about medication administration has been successful? a. The patient shakes the device before use. b. The patient attaches a spacer to the Diskus. c. The patient rapidly inhales the medication. d. The patient performs huff coughing after inhalation.

ANS: C The patient should inhale the medication rapidly. Otherwise the dry particles will stick to the tongue and oral mucosa and not get inhaled into the lungs. Advair Diskus is a dry powder inhaler; shaking is not recommended. Spacers are not used with dry powder inhalers. Huff coughing is a technique to move mucus into larger airways to expectorate. The patient should not huff cough or exhale forcefully after taking Advair in order to keep the medication in the lungs. DIF: Cognitive Level: Apply (application) REF: 574 TOP: Nursing Process: Evaluation MSC: NCLEX: Physiological Integrity

The nurse documents the vital signs for a patient admitted 2 days ago with gram-negative sepsis: temperature 101.2° F, blood pressure 90/56 mm Hg, pulse 92, respirations 34. Which action should the nurse take next? a. Give the scheduled IV antibiotic. b. Give the PRN acetaminophen (Tylenol). c. Obtain oxygen saturation using pulse oximetry. d. Notify the health care provider of the patient's vital signs.

ANS: C The patient's increased respiratory rate in combination with the admission diagnosis of gram-negative sepsis indicates that acute respiratory distress syndrome (ARDS) may be developing. The nurse should check for hypoxemia, a hallmark of ARDS. The health care provider should be notified after further assessment of the patient. Giving the scheduled antibiotic and the PRN acetaminophen will also be done, but they are not the highest priority for a patient who may be developing ARDS.

A patient seen in the asthma clinic has recorded daily peak flows that are 75% of the baseline. Which action will the nurse plan to take next? a. Increase the dose of the leukotriene inhibitor. b. Teach the patient about the use of oral corticosteroids. c. Administer a bronchodilator and recheck the peak flow. d. Instruct the patient to keep the next scheduled follow-up appointment.

ANS: C The patient's peak flow reading indicates that the condition is worsening (yellow zone). The patient should take the bronchodilator and recheck the peak flow. Depending on whether the patient returns to the green zone, indicating well-controlled symptoms, the patient may be prescribed oral corticosteroids or a change in dosing of other medications. Keeping the next appointment is appropriate, but the patient also needs to be taught how to control symptoms now and use the bronchodilator. DIF: Cognitive Level: Apply (application) REF: 580 TOP: Nursing Process: Planning MSC: NCLEX: Physiological Integrity

The nurse assesses a patient with a history of asthma. Which assessment finding indicates that the nurse should take immediate action? a. Pulse oximetry reading of 91% b. Respiratory rate of 26 breaths/minute c. Use of accessory muscles in breathing d. Peak expiratory flow rate of 240 L/minute

ANS: C Use of accessory muscle indicates that the patient is experiencing respiratory distress and rapid intervention is needed. The other data indicate the need for ongoing monitoring and assessment but do not suggest that immediate treatment is required. DIF: Cognitive Level: Apply (application) REF: 564-565 OBJ: Special Questions: Prioritization TOP: Nursing Process: Assessment MSC: NCLEX: Physiological Integrity

The nurse takes an admission history on a patient with possible asthma who has new-onset wheezing and shortness of breath. Which information may indicate a need for a change in therapy? a. The patient has chronic inflammatory bowel disease. b. The patient has a history of pneumonia 6 months ago. c. The patient takes propranolol (Inderal) for hypertension. d. The patient uses acetaminophen (Tylenol) for headaches.

ANS: C β-Blockers such as propranolol can cause bronchospasm in some patients with asthma. The other information will be documented in the health history but does not indicate a need for a change in therapy. DIF: Cognitive Level: Apply (application) REF: 576 TOP: Nursing Process: Assessment MSC: NCLEX: Physiological Integrity

The nurse is caring for a 10-month-old infant with respiratory syncytial virus (RSV) bronchiolitis. Which interventions should be included in the childs care (Select all that apply)? a. Administer antibiotics. b. Administer cough syrup. c. Encourage infant to drink 8 ounces of formula every 4 hours. d. Institute cluster care to encourage adequate rest. e. Place on noninvasive oxygen monitoring.

ANS: C, D, E Hydration is important in children with RSV bronchiolitis to loosen secretions and prevent shock. Clustering o care promotes periods of rest. The use of noninvasive oxygen monitoring is recommended. PTS: 1 DIF: Cognitive Level: Application REF: 1212 OBJ: Nursing Process: Implementation MSC:

Cystic fibrosis (CF) is suspected in a toddler. Which test is essential in establishing this diagnosis? a. Bronchoscopy b. Serum calcium c. Urine creatinine d. Sweat chloride test

ANS: D A sweat chloride test result greater than 60 mEq/L is diagnostic of CF. Although bronchoscopy is helpful for identifying bacterial infection in children with CF, it is not diagnostic. Serum calcium is normal in children with CF. Urine creatinine is not diagnostic of CF.

Skin testing for tuberculosis (the Mantoux test) is recommended: a. Every year for all children older than 2 years. b. Every year for all children older than 10 years. c. Every 2 years for all children starting at age 1 year. d. Periodically for children who reside in high-prevalence regions.

ANS: D Children who reside in high prevalence regions for tuberculosis should be tested every 2 to 3 years. Annual testing is not necessary. Testing is not necessary unless exposure is likely or an underlying medical risk factor is present.

Pancreatic enzymes are administered to the child with cystic fibrosis. Nursing considerations should include: a. Do not administer pancreatic enzymes if the child is receiving antibiotics. b. Decrease dose of pancreatic enzymes if the child is having frequent, bulky stools. c. Administer pancreatic enzymes between meals if at all possible. d. Pancreatic enzymes can be swallowed whole or sprinkled on a small amount of food taken at the beginning of a meal.

ANS: D Enzymes may be administered in a small amount of cereal or fruit or swallowed whole at the beginning of a meal, not between meals. Pancreatic enzymes are not contraindicated with antibiotics. The dose of enzymes should be increased if the child is having frequent, bulky stools. PTS: 1 DIF: Cognitive Level: Application REF: 1238 OBJ: Nursing Process: Implementation MSC:

Which statement is characteristic of acute otitis media (AOM)? a. The etiology is unknown. b. Permanent hearing loss often results. c. It can be treated by intramuscular antibiotics. d. It is treated with a broad range of antibiotics.

ANS: D Historically AOM has been treated with a range of antibiotics, and it is the most common disorder treated with antibiotics in the ambulatory setting. The etiology of AOM may be bacterial, such as Streptococcus pneumoniae, Haemophilus influenzae, and Moraxella catarrhalis, or a viral agent. Recent concerns about drug- resistant organisms have caused authorities to recommend judicious use of antibiotics and that antibiotics are not required for initial treatment. Permanent hearing loss is not a frequent cause of properly treated AOM. Intramuscular antibiotics are not necessary. Oral amoxicillin is the treatment of choice.

The parent of an infant with nasopharyngitis should be instructed to notify the health care professional if the infant: a. Becomes fussy. b. Has a cough. c. Has a fever over 99 F. d. Shows signs of an earache.

ANS: D If an infant with nasopharyngitis has a fever over 101 F, there is early evidence of respiratory complications. Irritability and a slight fever are common in an infant with a viral illness. Cough can be a sign of nasopharyngitis. PTS: 1 DIF: Cognitive Level: Application REF: 1200 OBJ: Nursing Process: Implementation MSC:

Which information should the nurse teach workers at a day care center about respiratory syncytial virus (RSV)? a. RSV is transmitted through particles in the air. b. RSV can live on skin or paper for up to a few seconds after contact. c. RSV can survive on nonporous surfaces for about 60 minutes. d. Frequent hand washing can decrease the spread of the virus.

ANS: D Meticulous hand washing can decrease the spread of organisms. RSV infection is not airborne. It is acquired mainly through contact with contaminated surfaces. RSV can live on skin or paper for up to 1 hour and on cribs and other nonporous surfaces for up to 6 hours.

Chronic otitis media with effusion (OME) is differentiated from acute otitis media (AOM) because it is usually characterized by: a. Fever as high as 40 C (104 F). b. Severe pain in the ear. c. Nausea and vomiting. d. A feeling of fullness in the ear.

ANS: D OME is characterized by an immobile or orange-discolored tympanic membrane and nonspecific complaints and does not cause severe pain. Fever and severe pain may be signs of AOM. Nausea and vomiting are associated with otitis media. PTS: 1 DIF: Cognitive Level: Comprehension REF: 1205 OBJ: Nursing Process: Diagnosis MSC:

Parents have understood teaching about prevention of childhood otitis media if they make which statement? a. We will only prop the bottle during the daytime feedings. b. Breastfeeding will be discontinued after 4 months of age. c. We will place the child flat right after feedings. d. We will be sure to keep immunizations up to date.

ANS: D Parents have understood the teaching about preventing childhood otitis media if they respond they will keep childhood immunizations up to date. The child should be maintained upright during feedings and after. Otitis media can be prevented by exclusively breastfeeding until at least 6 months of age. Propping bottles is discouraged to avoid pooling of milk while the child is in the supine position.

When caring for an infant with an upper respiratory tract infection and elevated temperature, an appropriate nursing intervention is to: a. Give tepid water baths to reduce fever. b. Encourage food intake to maintain caloric needs. c. Have child wear heavy clothing to prevent chilling. d. Give small amounts of favorite fluids frequently to prevent dehydration.

ANS: D Preventing dehydration by small frequent feedings is an important intervention in the febrile child. Tepid water baths may induce shivering, which raises temperature. Food should not be forced; it may result in the child vomiting. The febrile child should be dressed in light, loose clothing. PTS: 1 DIF: Cognitive Level: Application REF: 1199 OBJ: Nursing Process: Implementation MSC:

Why do infants and young children quickly have respiratory distress in acute and chronic alterations of the respiratory system? a. They have a widened, shorter airway. b. There is a defect in their sucking ability. c. The gag reflex increases mucus production. d. Mucus and edema obstruct small airways.

ANS: D The airway in infants and young children is narrower, not wider, and respiratory distress can occur quickly because mucus and edema can cause obstruction to their small airways. Sucking is not necessarily related to problems with the airway. The gag reflex is necessary to prevent aspiration. It does not produce mucus.

The mother of a toddler yells to the nurse, Help! He is choking to death on his food. The nurse determines that lifesaving measures are necessary based on: a. Gagging. b. Coughing. c. Pulse over 100 beats/min. d. Inability to speak.

ANS: D The inability to speak indicates a foreign-body airway obstruction of the larynx. Abdominal thrusts are needed for treatment of the choking child. Gagging indicates irritation at the back of the throat, not obstruction. Coughing does not indicate a complete airway obstruction. Tachycardia may be present for many reasons. PTS: 1 DIF: Cognitive Level: Application REF:

The nurse teaches a patient how to administer formoterol (Perforomist) through a nebulizer. Which action by the patient indicates good understanding of the teaching? a. The patient attaches a spacer before using the inhaler. b. The patient coughs vigorously after using the inhaler. c. The patient activates the inhaler at the onset of expiration. d. The patient removes the facial mask when misting has ceased.

ANS: D A nebulizer is used to administer aerosolized medication. A mist is seen when the medication is aerosolized, and when all of the medication has been used, the misting stops. The other options refer to inhaler use. Coughing vigorously after inhaling and activating the inhaler at the onset of expiration are both incorrect techniques when using an inhaler. DIF: Cognitive Level: Apply (application) REF: 574 TOP: Nursing Process: Evaluation MSC: NCLEX: Physiological Integrity

When assessing a patient with chronic obstructive pulmonary disease (COPD), the nurse finds a new onset of agitation and confusion. Which action should the nurse take first? a. Notify the health care provider. b. Check pupils for reaction to light. c. Attempt to calm and reorient the patient. d. Assess oxygenation using pulse oximetry.

ANS: D Because agitation and confusion are frequently the initial indicators of hypoxemia, the nurse's initial action should be to assess oxygen saturation. The other actions are also appropriate, but assessment of oxygenation takes priority over other assessments and notification of the health care provider.

A patient with acute respiratory distress syndrome (ARDS) who is intubated and receiving mechanical ventilation develops a right pneumothorax. Which action will the nurse anticipate taking next? a. Increase the tidal volume and respiratory rate. b. Increase the fraction of inspired oxygen (FIO2). c. Perform endotracheal suctioning more frequently. d. Lower the positive end-expiratory pressure (PEEP).

ANS: D Because barotrauma is associated with high airway pressures, the level of PEEP should be decreased. The other actions will not decrease the risk for pneumothorax.

Postural drainage with percussion and vibration is ordered twice daily for a patient with chronic bronchitis. Which intervention should the nurse include in the plan of care? a. Schedule the procedure 1 hour after the patient eats. b. Maintain the patient in the lateral position for 20 minutes. c. Perform percussion before assisting the patient to the drainage position. d. Give the ordered albuterol (Proventil) before the patient receives the therapy.

ANS: D Bronchodilators are administered before chest physiotherapy. Postural drainage, percussion, and vibration should be done 1 hour before or 3 hours after meals. Patients remain in each postural drainage position for 5 minutes. Percussion is done while the patient is in the postural drainage position. DIF: Cognitive Level: Apply (application) REF: 594 TOP: Nursing Process: Planning MSC: NCLEX: Physiological Integrity

The nurse develops a teaching plan to help increase activity tolerance at home for an older adult with severe chronic obstructive pulmonary disease (COPD). Which instructions would be most appropriate for the nurse to include in the plan of care? a. Stop exercising when short of breath. b. Walk until pulse rate exceeds 130 beats/minute. c. Limit exercise to activities of daily living (ADLs). d. Walk 15 to 20 minutes daily at least 3 times/week.

ANS: D Encourage the patient to walk 15 to 20 minutes a day at least three times a week with gradual increases. Shortness of breath is normal with exercise and not an indication that the patient should stop. Limiting exercise to ADLs will not improve the patient's exercise tolerance. A 70-year-old patient should have a pulse rate of 120 or less with exercise (80% of the maximal heart rate of 150). DIF: Cognitive Level: Apply (application) REF: 599 TOP: Nursing Process: Planning MSC: NCLEX: Physiological Integrity

Which nursing interventions included in the care of a mechanically ventilated patient with acute respiratory failure can the registered nurse (RN) delegate to an experienced licensed practical/vocational nurse (LPN/LVN) working in the intensive care unit? a. Assess breath sounds every hour. b. Monitor central venous pressures. c. Place patient in the prone position. d. Insert an indwelling urinary catheter.

ANS: D Insertion of indwelling urinary catheters is included in LPN/LVN education and scope of practice and can be safely delegated to an LPN/LVN who is experienced in caring for critically ill patients. Placing a patient who is on a ventilator in the prone position requires multiple staff, and should be supervised by an RN. Assessment of breath sounds and obtaining central venous pressures require advanced assessment skills and should be done by the RN caring for a critically ill patient.

The nurse completes an admission assessment on a patient with asthma. Which information given by patient is most indicative of a need for a change in therapy? a. The patient uses albuterol (Proventil) before any aerobic exercise. b. The patient says that the asthma symptoms are worse every spring. c. The patient's heart rate increases after using the albuterol (Proventil) inhaler. d. The patient's only medications are albuterol (Proventil) and salmeterol (Serevent).

ANS: D Long-acting β2-agonists should be used only in patients who also are using an inhaled corticosteroid for long-term control. Salmeterol should not be used as the first-line therapy for long-term control. Using a bronchodilator before exercise is appropriate. The other information given by the patient requires further assessment by the nurse, but is not unusual for a patient with asthma. DIF: Cognitive Level: Apply (application) REF: 572 TOP: Nursing Process: Assessment MSC: NCLEX: Physiological Integrity

A 55-year-old patient with increasing dyspnea is being evaluated for a possible diagnosis of chronic obstructive pulmonary disease (COPD). When teaching a patient about pulmonary function testing (PFT) for this condition, what is the most important question the nurse should ask? a. "Are you claustrophobic?" b. "Are you allergic to shellfish?" c. "Do you have any metal implants or prostheses?" d. "Have you taken any bronchodilators in the past 6 hours?"

ANS: D Pulmonary function testing will help establish the COPD diagnosis. Bronchodilators should be avoided at least 6 hours before the test. PFTs do not involve being placed in an enclosed area such as for magnetic resonance imaging (MRI). Contrast dye is not used for PFTs. The patient may still have PFTs done if metal implants or prostheses are present, as these are contraindications for an MRI. DIF: Cognitive Level: Apply (application) REF: 566 TOP: Nursing Process: Planning MSC: NCLEX: Physiological Integrity

A patient with chronic obstructive pulmonary disease (COPD) has a nursing diagnosis of impaired breathing pattern related to anxiety. Which nursing action is most appropriate to include in the plan of care? a. Titrate oxygen to keep saturation at least 90%. b. Discuss a high-protein, high-calorie diet with the patient. c. Suggest the use of over-the-counter sedative medications. d. Teach the patient how to effectively use pursed lip breathing.

ANS: D Pursed lip breathing techniques assist in prolonging the expiratory phase of respiration and decrease air trapping. There is no indication that the patient requires oxygen therapy or an improved diet. Sedative medications should be avoided because they decrease respiratory drive. DIF: Cognitive Level: Apply (application) REF: 578 TOP: Nursing Process: Planning MSC: NCLEX: Physiological Integrity

The nurse teaches a patient who has asthma about peak flow meter use. Which action by the patient indicates that teaching was successful? a. The patient inhales rapidly through the peak flow meter mouthpiece. b. The patient takes montelukast (Singulair) for peak flows in the red zone. c. The patient calls the health care provider when the peak flow is in the green zone. d. The patient uses albuterol (Proventil) metered dose inhaler (MDI) for peak flows in the yellow zone.

ANS: D Readings in the yellow zone indicate a decrease in peak flow. The patient should use short-acting β2-adrenergic (SABA) medications. Readings in the green zone indicate good asthma control. The patient should exhale quickly and forcefully through the peak flow meter mouthpiece to obtain the readings. Readings in the red zone do not indicate good peak flow, and the patient should take a fast-acting bronchodilator and call the health care provider for further instructions. Singulair is not indicated for acute attacks but rather is used for maintenance therapy. DIF: Cognitive Level: Apply (application) REF: 568 | 580 TOP: Nursing Process: Evaluation MSC: NCLEX: Physiological Integrity

A young adult patient with cystic fibrosis (CF) is admitted to the hospital with increased dyspnea. Which intervention should the nurse include in the plan of care? a. Schedule a sweat chloride test. b. Arrange for a hospice nurse visit. c. Place the patient on a low-sodium diet. d. Perform chest physiotherapy every 4 hours.

ANS: D Routine scheduling of airway clearance techniques is an essential intervention for patients with CF. A sweat chloride test is used to diagnose CF, but it does not provide any information about the effectiveness of therapy. There is no indication that the patient is terminally ill. Patients with CF lose excessive sodium in their sweat and require high amounts of dietary sodium. DIF: Cognitive Level: Apply (application) REF: 594 TOP: Nursing Process: Planning MSC: NCLEX: Physiological Integrity

A child has had cold symptoms for more than 2 weeks, a headache, nasal congestion with purulent nasal drainage, facial tenderness, and a cough that increases during sleep. The nurse recognizes that these symptoms are characteristic of which respiratory condition? a. Allergic rhinitis b. Bronchitis c. Asthma d. Sinusitis

ANS: D Sinusitis is characterized by signs and symptoms of a cold that do not improve after 14 days, a low-grade fever, nasal congestion and purulent nasal discharge, headache, tenderness, a feeling of fullness over the affected sinuses, halitosis, and a cough that increases when the child is lying down. The classic symptoms of allergic rhinitis are watery rhinorrhea; itchy nose, eyes, ears, and palate; and sneezing. Symptoms occur as long as the child is exposed to the allergen. Bronchitis is characterized by a gradual onset of rhinitis and a cough that is initially nonproductive but may change to a loose cough. The manifestations of asthma may vary, with wheezing being a classic sign. The symptoms presented in the question do not suggest asthma. PTS: 1 DIF: Cognitive Level: Application REF: 1201 OBJ: Nursing Process: Assessment MSC:

The nurse is caring for a 78-year-old patient who was hospitalized 2 days earlier with community-acquired pneumonia. Which assessment information is most important to communicate to the health care provider? a. Scattered crackles bilaterally in the posterior lung bases. b. Persistent cough that is productive of blood-tinged sputum. c. Temperature of 101.5° F (38.6° C) after 2 days of IV antibiotic therapy. d. Decreased oxygen saturation to 90% with 100% O2 by non-rebreather mask.

ANS: D The patient's low SpO2 despite receiving a high fraction of inspired oxygen (FIO2) indicates the possibility of acute respiratory distress syndrome (ARDS). The patient's blood-tinged sputum and scattered crackles are not unusual in a patient with pneumonia, although they do require continued monitoring. The continued temperature elevation indicates a possible need to change antibiotics, but this is not as urgent a concern as the progression toward hypoxemia despite an increase in O2 flow rate.

A patient with severe chronic obstructive pulmonary disease (COPD) tells the nurse, "I wish I were dead! I'm just a burden on everybody." Based on this information, which nursing diagnosis is most appropriate? a. Complicated grieving related to expectation of death b. Ineffective coping related to unknown outcome of illness c. Deficient knowledge related to lack of education about COPD d. Chronic low self-esteem related to increased physical dependence

ANS: D The patient's statement about not being able to do anything for himself or herself supports this diagnosis. Although deficient knowledge, complicated grieving, and ineffective coping may also be appropriate diagnoses for patients with COPD, the data for this patient do not support these diagnoses. DIF: Cognitive Level: Apply (application) REF: 599-600 TOP: Nursing Process: Diagnosis MSC: NCLEX: Psychosocial Integrity

A patient hospitalized with chronic obstructive pulmonary disease (COPD) is being discharged home on oxygen therapy. Which instruction should the nurse include in the discharge teaching? a. Storage of oxygen tanks will require adequate space in the home. b. Travel opportunities will be limited because of the use of oxygen. c. Oxygen flow should be increased if the patient has more dyspnea. d. Oxygen use can improve the patient's prognosis and quality of life.

ANS: D The use of home oxygen improves quality of life and prognosis. Because increased dyspnea may be a symptom of an acute process such as pneumonia, the patient should notify the physician rather than increasing the oxygen flow rate if dyspnea becomes worse. Oxygen can be supplied using liquid, storage tanks, or concentrators, depending on individual patient circumstances. Travel is possible using portable oxygen concentrators. DIF: Cognitive Level: Apply (application) REF: 592 TOP: Nursing Process: Implementation MSC: NCLEX: Physiological Integrity

After receiving change-of-shift report on a medical unit, which patient should the nurse assess first? a. A patient with cystic fibrosis who has thick, green-colored sputum b. A patient with pneumonia who has crackles bilaterally in the lung bases c. A patient with emphysema who has an oxygen saturation of 90% to 92% d. A patient with septicemia who has intercostal and suprasternal retractions

ANS: D This patient's history of septicemia and labored breathing suggest the onset of ARDS, which will require rapid interventions such as administration of oxygen and use of positive pressure ventilation. The other patients should also be assessed as quickly as possible, but their assessment data are typical of their disease processes and do not suggest deterioration in their status.

Which information will the nurse include in the asthma teaching plan for a patient being discharged? a. Use the inhaled corticosteroid when shortness of breath occurs. b. Inhale slowly and deeply when using the dry powder inhaler (DPI). c. Hold your breath for 5 seconds after using the bronchodilator inhaler. d. Tremors are an expected side effect of rapidly acting bronchodilators.

ANS: D Tremors are a common side effect of short-acting β2-adrenergic (SABA) medications and not a reason to avoid using the SABA inhaler. Inhaled corticosteroids do not act rapidly to reduce dyspnea. Rapid inhalation is needed when using a DPI. The patient should hold the breath for 10 seconds after using inhalers. DIF: Cognitive Level: Apply (application) REF: 572 TOP: Nursing Process: Implementation MSC: NCLEX: Physiological Integrity

A nurse is assigned to care for a client with bronchial irritation. The client is prescribed diphenhydramine. Before administering the drug, which of the following would the nurse do? A) Document color and amount of any sputum present. B) Record the previous prescriptions. C) Take vital signs every 4 hours. D) Assess the client's cardiovascular status.

Ans: A Feedback: Before drug administration, the nurse should document the color and amount of any sputum present. The nurse need not record the previous prescriptions; however, the nurse should determine if any drugs the client uses would potentially interact with diphenhydramine. The nurse needs to take the client's vital signs, but not every 4 hours. The nurse needs to assess the respiratory status of the client before administering mucolytics and expectorants, but not before administering diphenhydramine HCl. Assessing the client's cardiovascular status is not necessary.

Which of the following is most likely to precipitate an asthmatic attack in a child with a diagnosis of extrinsic, or atopic, asthma? A) Pet dander B) Cold weather C) Stress D) Respiratory tract infections

Ans: A Feedback: Extrinsic or atopic asthma is typically initiated by a type I hypersensitivity reaction induced by exposure to an extrinsic antigen or allergen such as pet dander. Intrinsic or nonatopic asthma triggers include respiratory tract infections, exercise, hyperventilation, cold air, drugs and chemicals, hormonal changes and emotional upsets, airborne pollutants, and gastroesophageal reflux.

Which of the following individuals is experiencing an immunologic lung disorder affecting ventilation that has caused the formation of a granuloma on chest x-ray? A) A 30-year-old African American man who has been diagnosed with sarcoidosis B) An infant whose routine screening is suggestive of cystic fibrosis C) An elderly, lifelong smoker who has been admitted to hospital with emphysema exacerbation D) A 16-year-old girl who must limit her physical activity to prevent the onset of acute asthmatic attacks

Ans: A Feedback: Sarcoidosis is an example of restrictive lung disease, whereas cystic fibrosis (CF), emphysema, and asthma are considered obstructive.

A 51-year-old man has been diagnosed with chronic bronchitis after a long history of recurrent coughing. Which of the man's following statements demonstrates a sound understanding of his new diagnosis? A) "If I had quit smoking earlier than I did, I think I could have avoided getting bronchitis." B) "I'm pretty sure that I first caught bronchitis from the person who has the cubicle next to mine at work." C) "I read on the Internet that I might have got bronchitis because I was born with an enzyme deficiency." D) "I think that I probably could have prevented this if I had got in the habit of exercising more when I was younger."

Ans: A Feedback: Smoking is frequently implicated in the etiology of chronic bronchitis. Infections do not typically initiate the disease, and exercise is not noted to have preventative value. Enzyme deficiency is associated with emphysema, but not bronchitis.

A client with asthma has been prescribed an antiasthmatic drug. Before administering the drug, the nurse assesses the respiratory rate of the client. The nurse notifies the primary health care provider based on which finding? A) 10 breaths/min B) 14 breaths/min C) 18 breaths/min D) 22 breaths/min

Ans: A Feedback: The nurse should consider 10 breaths/min an abnormal respiratory rate and notify the primary health care provider. Respiratory rates below 12 breaths/min or above 24 breaths/min are considered abnormal. A respiratory rate between 12 breaths/min and 24 breaths/min is considered normal.

A client is prescribed albuterol for bronchospasm in chronic bronchial asthma. Which of the following nursing diagnoses would the nurse expect to see on the care plan as a result of the adverse reaction of albuterol? A) Anxiety B) Risk of Impaired Oral Mucous Membranes C) Ineffective Tissue Perfusion D) Risk of Injury

Ans: A Feedback: The nurse would most likely identify a nursing diagnosis of Anxiety related to the adverse reaction of albuterol. A nursing diagnosis of Risk of Impaired Mucous Membranes may be seen with the use of corticosteroids, which increase the risk of oral candidiasis. There is no increased risk of injury or ineffective tissue perfusion with the use of albuterol therapy.

A young, male child is born with severe respiratory failure. Over the course of months, the parents note his body looks swollen. They ask, "Is our baby's kidneys not working right? Why is he so swollen?" The nurse bases his or her reply on which of the following physiological principles? A) "The right side of his heart (cor pulmonale) is not pumping effectively. Blood is backlogging in his body, which is why he is so swollen." B) "We just need to call the physician and ask him to give you a prescription for more water pills." C) "Once we get his oxygenation level back to normal, then maybe his kidneys will receive enough oxygenated blood to filter better." D) "This happens when he has so many secretions in his lungs. Maybe we should try some expectorant to thin his secretions so he can cough them out."

Ans: A Feedback: The term cor pulmonale refers to right heart failure resulting from primary lung disease or pulmonary hypertension. The increased pressures and work result in hypertrophy and eventual failure of the right ventricle. The manifestations of cor pulmonale include signs of right-sided heart failure, which include venous congestion, peripheral edema, shortness of breath, and a productive cough, which becomes worse during periods of heart failure. None of the other statements are applicable to these parents' questions.

A nurse is caring for a client with thick sputum who is having difficulty bringing up mucus. Which nursing diagnosis would the nurse most likely identify? A) Ineffective Airway Clearance B) Acute Pain C) Risk for Injury D) Impaired Oral Mucous Membranes

Ans: A Feedback: Thick sputum interferes with moving air effectively in and out of the respiratory tract. Therefore, the most likely nursing diagnosis would be Ineffective Airway Clearance. There is no evidence of pain. Risk for Injury would be appropriate if the client was experiencing sedation or drowsiness from the prescribed medication. Impaired Oral Mucous Membranes would be appropriate if the client was experiencing dry mouth from the medication.

Which of the following clients is at risk for developing acute respiratory failure? A) A middle-aged male diagnosed with amyotrophic lateral sclerosis (ALS) who has pneumonia with low O2 saturation B) An elderly female living in senior housing who has been exposed to a "cold" while her grandchildren visited C) A teenager in a high school that has had an increase in student absences due to an outbreak of strep throat D) A toddler in daycare who has been sharing toys with peers before the staff could sanitize properly

Ans: A Feedback: Acute respiratory failure may occur in previously healthy persons as the result of acute disease or trauma involving the respiratory system, or it may develop in the course of a chronic neuromuscular or lung disease. ALS is a neurodegenerative disease with various causes. It is characterized by rapidly progressive weakness due to muscle atrophy and muscle spasticity and difficulty in speaking (dysarthria), swallowing (dysphagia), and breathing (dyspnea). A common cold, strep throat, or sharing toys in daycare will not necessarily place one at risk for respiratory failure.

An elderly client who has been restricted to bed by numerous comorbidities for several weeks has been diagnosed with a large pleural effusion. Which of the following treatment modalities is most likely to resolve the client's most recent health problem? A) Thoracentesis B) Supplementary oxygen therapy C) Administration of corticosteroids D) Administration of bronchodilators

Ans: A Feedback: With large effusions, thoracentesis may be used to remove fluid from the intrapleural space and allow for reexpansion of the lung.

A client is prescribed an inhaled corticosteroid. The nurse would instruct the client about which of the following as a possible adverse reaction? Select all that apply. A) Fungal infection B) Pharyngeal irritation C) Blurred vision D) Bradycardia E) Insomnia

Ans: A, B Feedback: Adverse reactions of inhaled corticosteroids include oral, laryngeal, and pharyngeal irritation and fungal infection. Blurred vision, bradycardia, and insomnia are not associated with inhaled corticosteroids.

A client suffers from motion sickness. Which of the following would the nurse anticipate the primary health care provider to prescribe? Select all that apply. A) Promethazine (Phenergan) B) Diphenhydramine (Benadryl) C) Levocetirizine (Xyzal) D) Azelastine (Astelin) E) Pseudoephedrine (Sudafed)

Ans: A, B Feedback: Both promethazine and diphenhydramine can be used in the treatment of motion sickness. None of the other drugs listed would be appropriate.

A nursing instructor is describing the advantages of using a second-generation antihistamine over a first-generation antihistamine. Which of the following would the instructor most likely include? Select all that apply. A) Less sedation B) Fewer anticholinergic effects C) Less nausea D) Can be used during pregnancy E) Can be used during lactation

Ans: A, B Feedback: Second-generation antihistamines cause less sedation and fewer anticholinergic effects because they selectively bind to peripheral rather than central H1 receptors. Some first-generation antihistamines can be used as antiemetics. Use of any antihistamine is contraindicated during pregnancy and lactation.

Based on the nurse's understanding of decongestants, the nurse would expect to administer this drug cautiously to a client with which of the following? Select all that apply. A) Diabetes B) Glaucoma C) Hypotension D) Hypothyroidism E) Arthritis

Ans: A, B Feedback: The nurse should administer decongestants cautiously to clients with diabetes, heart disease, hypertension, hyperthyroidism, benign prostatic hypertrophy, and glaucoma. These clients should contact their primary health care provider before taking over-the-counter decongestants.

The nurse understands that codeine-containing antitussives should be used cautiously in clients with which of the following conditions? Select all that apply. A) Pregnancy B) Convulsive disorders C) Prostatic hypertrophy D) Hyperlipidemia E) Type 2 diabetes

Ans: A, B, C Feedback: Codeine-containing antitussives should be used cautiously during pregnancy and labor and in clients with COPD, acute asthma attacks, pre-existing respiratory disorders, acute abdominal conditions, head injury, increased intracranial pressure, convulsive disorders, hepatic or renal impairment, and prostatic hypertrophy.

After teaching a group of nursing students about antihistamines, the instructor determines that the teaching was successful when the students identify which of the following as less sedating? Select all that apply. A) Loratadine (Claritin) B) Fexofenadine (Allegra) C) Cetirizine (Zyrtec) D) Brompheniramine (Lodrane) E) Clemastine (Tavist)

Ans: A, B, C Feedback: Loratadine, fexofenadine, and cetirizine are considered second-generation antihistamines that are less sedating than first-generation antihistamines (brompheniramine and clemastine).

Eucalyptus is an herbal product that can be used as a decongestant and expectorant. In which of the following client populations would the nurse identify a contraindication for its use? Select all that apply. A) Pregnant females B) Children younger than 2 years of age C) Lactating females D) Postmenopausal females E) Men

Ans: A, B, C Feedback: The use of eucalyptus is contraindicated during pregnancy and lactation, as well as in people who are hypersensitive to eucalyptus and in children younger than 2 years of age.

Before administering an antitussive to a client, which of the following would the nurse assess? Select all that apply. A) Temperature B) Sputum presence C) Type of cough D) Bowel sounds E) Heart sounds

Ans: A, B, C Feedback: Vital signs, type of cough, presence of sputum, color and amount of sputum, home remedies used, and actions taken should be assessed prior to initiation of an antitussive. Bowel and heart sounds are not part of the preadministration assessment.

A client is prescribed zileuton (Zyflo). The nurse instructs the client to contact the primary health care provider if which of the following occur? Select all that apply. A) Jaundice B) Pruritus C) Fatigue D) Dizziness E) Restlessness

Ans: A, B, C Feedback: Zileuton may cause liver damage, which may present with the following symptoms: upper right quadrant pain, nausea, fatigue, lethargy, pruritus, and jaundice.

A pediatric client is prescribed cromolyn. The nurse understands that this drug can be administered in which manner? Select all that apply. A) Via a nebulizer B) Orally C) Nasal spray D) Metered-dose inhaler E) Subcutaneous injection

Ans: A, B, C, D Feedback: Cromolyn may be administered via a nebulizer, as an aerosol metered spray, as a nasal spray, or orally. It is not given subcutaneously.

A client with chronic asthma comes to the clinic for a follow-up visit. A nurse should question the client about which of the following? Select all that apply. A) Allergies B) Frequency of attacks C) Severity of attacks D) Antiasthma drugs currently being taken E) Antiasthma drugs taken in the past

Ans: A, B, C, D, E Feedback: In clients with chronic asthma, the nurse questions the client concerning allergies, frequency and severity of attacks, factors that cause or relieve attacks, and any antiasthma drugs used currently or taken previously.

A client with a history of heart failure and COPD (caused by 60 pack/year smoking) presents to the clinic with the following complaints: auscultation of breath sounds reveal absent/diminished breath sounds in the right lower lobe. Which other manifestations lead the health care provider to suspect the client may have developed atelectasis? Select all that apply. A) Respiratory rate—32; pulse rate—122 beats/minute. B) "Having a hard time catching my breath." C) "Seems like I'm not making much water (decreased urine production)." D) Using accessory muscles to help him breathe. E) Copious amounts of thick, green sputum.

Ans: A, B, D Feedback: Atelectasis is caused most commonly by airway obstruction rather than a vascular obstruction. The clinical manifestations of atelectasis include tachypnea (respiratory rate of 32), tachycardia (pulse rate of 122) dyspnea (hard time catching breath), cyanosis, signs of hypoxemia, diminished chest expansion, absence of breath sounds, and intercostal retractions (use of accessory muscles). Both chest expansion and breath sounds are decreased on the affected side. There may be intercostal retraction (pulling in of the intercostal spaces) over the involved area during inspiration. Urine production is not related to atelectasis. Copious green sputum is associated with infection.

A client has a nursing diagnosis of Ineffective Airway Clearance. Which of the following would the nurse include in the client's plan of care? Select all that apply. A) Encouraging increased fluid intake B) Assisting the client in taking deep, diaphragmatic breaths C) Discouraging client movement D) Instructing the client to avoid coughing E) Encouraging the client to change positions

Ans: A, B, E Feedback: Clients should be encouraged to change positions frequently, breathe deeply, and increase fluid intake to aid in effectively clearing the airway of sputum. Coughing helps to move mucus.

When assessing an older client who is receiving an antihistamine, the nurse integrates knowledge of which of the following as contributing to the client's higher risk of injury? Select all that apply. A) Hearing loss B) Steady gait C) Visual impairment D) Hypertension E) Diabetes

Ans: A, C Feedback: Older adult clients are more likely to experience injury from dizziness because with age comes an increased risk for falls due to sensorimotor deficits (hearing loss, visual impairment) or unsteady gait.

Before administering an antihistamine to a client, which of the following would the nurse include in the preadministration assessment? Select all that apply. A) Asking about symptoms B) Checking visual acuity C) Asking about prescription medications D) Auscultating bowel sounds E) Auscultating heart sounds

Ans: A, C Feedback: The preadministration assessment for clients receiving antihistamines depends on the reason for use but should at the least include asking about symptoms of the involved areas and asking about prescription medications the client is taking.

While lecturing on COPD, the instructor mentions emphysema. The instructor asks the students, "If the client is a smoker, explain the physiology behind cellular changes occurring in the lung which allow destruction of the alveoli." Which students have an accurate response? Select all that apply. A) "Antiprotease production and release is not adequate to neutralize the excess protease production." B) "The capillary beds can no longer bring the cells to the lung to fight off the infection since they are blocked by fatty plaque." C) "There is 1-antitrypsin deficiency, so this enzyme can't protect the lung from damage." D) "The alveolar tissue is being digested by enzymes; therefore, they can't grow back to restore normal ventilation."

Ans: A, C Feedback: Emphysema is characterized by a loss of lung elasticity and abnormal enlargement of the air spaces distal to the terminal bronchioles, with destruction of the alveolar walls. One of the recognized causes of emphysema is an inherited deficiency of 1-antitrypsin, an antiprotease enzyme that protects the lung from injury. Cigarette smoke stimulates the movement of inflammatory cells into the lungs, resulting in increased release of elastase and other proteases. In smokers in whom COPD develops, antiprotease production and release may be inadequate to neutralize the excess protease production such that the process of elastic tissue destruction goes unchecked.

Before administering a prescribed bronchodilator to a client experiencing acute breathing distress, which of the following would be appropriate for the nurse to assess? Select all that apply. A) Blood pressure B) Blood glucose C) Pulse D) Lung sounds E) Respiratory rate

Ans: A, C, D, E Feedback: Prior to initiation of a bronchodilator during acute breathing distress, the nurse needs to assess vital signs, including blood pressure, pulse, and respiratory rate and lung sounds.

A client who is experiencing an acute asthmatic attack receives prescribed therapy with a bronchodilator. As part of the plan of care, the nurse would continue to assess which of the following every 4 hours? Select all that apply. A) Intake B) Blood pressure C) Output D) Lung sounds E) Accessory muscle use

Ans: A, C, D, E Feedback: The nurse should note the client's respiratory rate, lungs sounds, intake, and output and use of accessory muscles in breathing every 4 hours during an acute asthma attack.

The nurse would be especially alert for the development of which of the following when administering an antihistamine to an older adult? Select all that apply. A) Hypotension B) Hypertension C) Dry mouth D) Insomnia E) Sedation

Ans: A, C, E Feedback: Older adult clients are more likely to experience anticholinergic effects (dry mouth), dizziness, sedation, hypotension, and confusion while taking an antihistamine.

A toddler seems to have a little "cold" and runny nose. At bedtime, he appears to be OK. A few hours later, parents awaken hearing a "tight" coughing sound. They recognize the child is not breathing well, so they rush to the emergency department. On arrival, the nurses suspect bronchial asthma based on which of the following assessment data? Select all that apply. A) Audible wheezing B) "Crowing sound with inspiration" C) Respiratory rate—44 with prolonged exhalation D) Coughing up blood-tinged sputum E) Sitting upright, leaning forward, and using accessory muscles to breathe

Ans: A, C, E Feedback: Bronchial asthma represents a reversible form of obstructive airway disease caused by narrowing of airways due to bronchospasms, inflammation, and increased airway secretions. Healthy children develop what may seem to be a cold with rhinorrhea, rapidly followed by irritability, a tight and nonproductive cough, wheezing, tachypnea (respiratory rate—44), dyspnea with prolonged expiration, and use of accessory muscles of respiration. Cyanosis, hyperinflation of the chest, and tachycardia indicate increasing severity of the asthma attack. Croup is characterized by inspiratory stridor or a barking cough. The British use the term croup to describe the cry of the crow or raven, and this is undoubtedly how the term originated. Hemoptysis (blood in the sputum) is usually associated with pulmonary emboli or lung cancers to name a few.

When describing the actions of upper respiratory system drugs, a nursing instructor explains that which of the following exerts its effect by increasing the production of secretions, thereby decreasing the viscosity? Select all that apply. A) Benzonatate B) Guaifenesin C) Codeine D) Potassium iodide E) Dextromethorphan

Ans: A, D Feedback: Benzonatate and potassium iodide are expectorants, which increase the production of secretions, thus making the secretions less viscous. Guaifenesin, codeine, and dextromethorphan are antitussives, which suppress cough.

A client is receiving theophylline. The nurse checks the client's serum theophylline level and finds it to be less than therapeutic. The nurse reviews the client's medical record, noting that which of the following might be a reason for this? Select all that apply. A) Nicotine B) Allopurinol C) Verapamil D) Phenytoin E) Ketoconazole

Ans: A, D, E Feedback: Nicotine, phenytoin (a hydantoin), and ketoconazole can decrease theophylline levels. Verapamil (a calcium channel blocker) and allopurinol can increase theophylline levels.

A client is suffering from rebound congestion. Which of the following would the nurse expect to do to help the client experience relief from rebound congestion? Select all that apply. A) Suggest the client switch from a topical decongestant to an oral product. B) Recommend the client switch from an oral decongestant to a topical product. C) Tell the client to abruptly discontinue the decongestant product. D) Advise the client to gradually discontinue the decongestant product. E) Suggest a saline irrigation of the nasal passages in place of the decongestant.

Ans: A, D, E Feedback: Rebound congestion can be treated by a switch from a topical to an oral decongestant, gradual discontinuation of the topical decongestant, or replacement of the topical decongestant with saline irrigation of the nasal passages.

After teaching a group of nursing students about xanthine derivatives, the instructor determines that the teaching was successful when the students state which of the following? Select all that apply. A) Xanthine derivatives cause flushing. B) The action of xanthine derivatives leads to bradycardia. C) Xanthine derivatives cause a reduction in airway inflammation. D) Xanthine derivatives cause hypoglycemia. E) Xanthine derivatives stimulate the CNS to promote bronchodilation.

Ans: A, E Feedback: Xanthine derivatives can cause flushing, tachycardia, and hyperglycemia. Xanthine derivatives elicit their effects by stimulating the CNS to promote bronchodilation.

Soon after delivery, the mother grabs the nurses arm and states, "Something's wrong...I can't get my breath." Which of the following assessments lead the nurse to suspect the client has had an amniotic emboli travel to the lungs? Select all that apply. A) BP 90/65; pulse 130, irregular; respiratory rate 35, shallow. B) Intercostal traction noted on inspiration. C) +3 pitting edema in lower extremities. D) Trachea has shifted and is no longer midline. E) Productive cough with blood-streaked sputum.

Ans: A, E Feedback: The embolism may consist of amniotic fluid that has entered the maternal circulation during childbirth. Persons with moderate-sized emboli often present with breathlessness accompanied by pleuritic pain, apprehension, slight fever, and cough productive of blood-streaked sputum. Tachycardia often occurs to compensate for decreased oxygenation, and the breathing pattern is rapid and shallow. Intercostal traction is usually associated with atelectasis; pitting edema is a sign of right-sided heart failure; trachea shift is a classic sign of a tension pneumothorax.

A group of nursing students are reviewing information about mast cell stabilizers. The students demonstrate understanding of the information when they identify which of the following as an example? A) Beclomethasone B) Cromolyn C) Albuterol D) Montelukast

Ans: B Feedback: Cromolyn is an example of a mast cell stabilizer. Beclomethasone is an inhaled corticosteroid. Albuterol is a short-acting beta-2 agonist. Montelukast is an example of a leukotriene modifier.

When evaluating the pulmonary function test results for a COPD client, which one correlates to the mismatch of ventilation and perfusion associated with this diagnosis? A) Forced vital capacity (FVC) is elevated. B) Forced expiratory volume (FEV) is decreased. C) Total lung capacity (TLC) is decreased. D) Marked decrease in residual volume (RV).

Ans: B Feedback: In clients with chronic lung disease, the FVC is decreased, the FEV1.0 is decreased, and the ratio of FEV1.0 to FVC is decreased. Lung volume measurements reveal a marked increase in RV, an increase in Total Lung Cc, and elevation of the RV-to-TLC ratio.

A client receives a loading dose of theophylline to treat acute respiratory symptoms. When assessing the client, the nurse would immediately notify the primary health care provider for which of the following? A) Constipation B) Abdominal cramps C) Bradycardia D) Mental depression

Ans: B Feedback: It is important for the nurse to closely monitor the client for signs of theophylline toxicity. The nurse should notify the primary health care provider immediately if any of the following signs of theophylline toxicity develop: anorexia, nausea, vomiting, diarrhea, confusion, abdominal cramping, headache, restlessness, insomnia, tachycardia, arrhythmias, or seizures. Constipation, bradycardia, and mental depression are not signs of theophylline toxicity.

After teaching a client about his upper respiratory drug therapy, the nurse determines that additional teaching is needed when the client identifies which of the following as a reason to notify his primary health care provider? A) Cough changes from nonproductive to productive. B) Sputum appears clear. C) Sputum increases. D) Shortness of breath occurs.

Ans: B Feedback: The client should notify his primary health care provider if the type of cough changes, sputum changes color or increases, and shortness of breath occurs. Clear sputum is normal.

A client has been prescribed a decongestant drug for congestion associated with rhinitis. When teaching the client about this drug, which of the following would the nurse include as a possible adverse reaction? A) Decreased pulse rate B) Blurred vision C) Drowsiness D) Dryness of throat

Ans: B Feedback: The nurse should inform the client that blurred vision is a possible adverse reaction of decongestant drugs. Additionally, an increased and not decreased pulse rate may also be seen. Drowsiness is not seen with decongestant usage. Dryness of the nasal mucosa and not the throat may be seen with decongestant drugs, which are used mostly as topical sprays and drops.

A client with a nonproductive cough has been prescribed dextromethorphan HBr LiquiCaps. What instructions should the nurse provide the client to promote an optimal response to therapy? A) Take the drug with a glass of milk. B) Swallow the whole tablet and do not chew it. C) Dissolve the tablet in water and take the drug. D) Take the drug on an empty stomach.

Ans: B Feedback: The nurse should instruct the client to swallow the whole tablet and not to chew it to ensure that the drug is absorbed properly. The nurse need not instruct the client to take the drug with a glass of milk, dissolve the tablet in water and take the drug, or take the drug on an empty stomach.

A client has been prescribed a nasal decongestant for nasal stuffiness due to a common cold. The client is also taking an antidepressant that is a monoamine oxidase inhibitor. The nurse would warn the client about which of the following? A) Hypotension B) Severe headache C) Sedation D) Bradycardia

Ans: B Feedback: The nurse should warn the client of the possibility of severe headache due to an interaction between the two drugs. Such an interaction may also result in hypertensive crisis instead of hypotension. Sedation and bradycardia do not occur when an MAOI and decongestant are used together.

A client who is receiving aminophylline complains of heartburn. Which of the following instructions should the nurse provide the client to help alleviate the condition? A) Eat small, frequent meals. B) Raise the head of the bed. C) Limit fluid intake with meals. D) Use strict oral hygiene.

Ans: B Feedback: When a client receiving aminophylline complains of heartburn, the nurse should instruct the client to remain upright with the head end of the bed raised. Eating small, frequent meals and limiting fluid intake with meals help alleviate the symptoms of nausea, and not of heartburn. Using strict oral hygiene helps prevent infection with Candida albicans seen with corticosteroid therapy.

Which of the following medications helps treat the inflammatory reaction of asthma clients diagnosed with late-phase asthma response? A) Anticholinergic agents B) Systemic corticosteroids C) Long-acting 2-agonists D) Phosphodiesterase inhibitors

Ans: B Feedback: A short course of systemic corticosteroids, administered orally or parenterally, may be used for treating the inflammatory reaction associated with the late-phase response. The anticholinergic agents block cholinergic receptors and reduce intrinsic vagal tone that causes bronchoconstriction. The long-acting 2 agonists, available for administration by the inhaled or oral routes, act by relaxing bronchial smooth muscle. Theophylline, a phosphodiesterase inhibitor, is a bronchodilator that acts by relaxing bronchial smooth muscle.

A teenaged cystic fibrosis client presents to the clinic. The health care provider (HCP) knows that cystic fibrosis (CF) causes severe chronic respiratory disease in children. In addition, the HCP should also focus his or her assessment on which of the other body systems affected by CF? A) Renal B) Pancreatic C) Cardiac D) Central nervous system

Ans: B Feedback: Cystic fibrosis (CF) is manifested by pancreatic exocrine deficiency and elevation of sodium chloride in the sweat. Cystic fibrosis (CF) is an inherited disorder involving fluid secretion by the exocrine glands in the epithelial lining of the respiratory, gastrointestinal, and reproductive tracts. Excessive loss of sodium in the sweat predisposes young children to salt depletion episodes. Respiratory manifestations are caused by an accumulation of viscid mucus in the bronchi, impaired mucociliary clearance, lung infections, bronchiectasis, and dilatation. The renal, cardiac, and CNS are usually not involved with CF manifestation.

A man sustained a puncture injury to his chest that caused a tension pneumothorax to form. This is a life-threatening condition because: A) Expired air exits the bleeding wou B) Trapped, inspired air collapses the lung. C) The opposite lung hyperinflates. D) Blebs on the lung surface rupture.

Ans: B Feedback: Tension pneumothorax occurs when the intrapleural pressure exceeds atmospheric pressure. It is a life-threatening condition and occurs when injury to the chest or respiratory structures permits air to enter but not leave the pleural space. Spontaneous pneumothorax occurs when an air-filled bleb, or blister, on the lung surface ruptures. Rupture of these blebs allows atmospheric air from the airways to enter the pleural cavity. This results in a rapid increase in pressure in the chest with a compression atelectasis of the unaffected lung.

Assessment of a client reveals that he is taking phenelzine (Nardil), a monoamine oxidase inhibitor, for depression. The client reports that he just started using dextromethorphan over the counter without consulting his primary health care provider. The nurse would be alert for which of the following? Select all that apply. A) Hypertension B) Fever C) Coma D) Constipation E) Shortness of breath

Ans: B, C Feedback: Coadministration of dextromethorphan and a monoamine oxidase inhibitor may result in hypotension, fever, nausea, leg jerking, and coma.

Before leaving the hospital after an acute asthma attack, a client is given a prescription for fluticasone/salmeterol (Advair) 250/50 to inhale one puff twice a day. The nurse completing the client's discharge teaching should tell the client which of the following? Select all that apply. A) Take the medication as needed. B) Continue to carry a rescue inhaler. C) Check peak flow daily. D) Rinse mouth after each use. E) Shake meter well before using.

Ans: B, C, D Feedback: The drug is a dry powder inhaler that contains an inhaled corticosteroid and a long-acting beta agonist. The medication should be taken every day as per the directions on the label to prevent future exacerbations. The client should continue to carry a rescue inhaler and check peak flow around the same time each day. Advair does not need to be shaken prior to use as it is a dry powder inhaler. The client should be advised to rinse his mouth out after each use to prevent oral thrush.

After administering a decongestant, which of the following would the nurse include in the ongoing assessment? Select all that apply. A) Heart sounds B) Blood pressure C) Level of congestion D) Therapeutic effect E) Pain assessment

Ans: B, C, D Feedback: The ongoing assessment for a client taking a decongestant should include assessment of blood pressure and pulse and asking about level of congestion, therapeutic effect, and adverse reactions.

The nurse teaches a client receiving an inhaled corticosteroid about the possibility of developing oral thrush. Which of the following would the nurse include in the teaching plan as a way to reduce this risk? Select all that apply. A) Need to avoid eating after administration B) Performing strict oral hygiene C) Cleaning the inhaler per package instructions D) Using proper technique when administering dose E) Administering a dose only every other day

Ans: B, C, D Feedback: To decrease the likelihood of developing oral thrush, a client should use strict oral hygiene, cleanse the inhaler as directed in the package instructions, and use proper technique when administering a dose. There is no need to avoid eating after administration, and using the drug only every other day would not be effective.

A client is demonstrating how to use a peak flow meter. The nurse determines that the client is successful when he does which of the following? Select all that apply. A) Inhales as forcibly as possible B) Stands upright to allow the best inhalation possible C) Makes sure indicator is at lowest level on scale D) Makes sure lips are sealed tightly around the mouthpiece E) Measures peak flow rate at different times each day

Ans: B, C, D Feedback: When teaching a client to use a peak flow meter, instruct the client to make sure the indicator is at the lowest level, stand upright, make sure the lips form a tight seal around the mouthpiece, exhale as forcibly and as quickly as possible, and test peak flow at the same time each day.

The nurse is reviewing the history of a client who is prescribed a long-acting beta-2 agonist. Which of the following would alert the nurse to the need to administer this drug cautiously to the client? Select all that apply. A) Hyperlipidemia B) Hypertension C) Glaucoma D) Hyperthyroidism E) Diabetes

Ans: B, C, D, E Feedback: Long-acting beta-2 agonists should be used cautiously in clients with hypertension, cardiac dysfunction, hyperthyroidism, glaucoma, diabetes, prostatic hypertrophy, and history of seizures.

After administering an antitussive, the nurse would continue to assess which of the following? Select all that apply. A) Heart sounds B) Lung sounds C) Frequency of cough D) Therapeutic effect E) Pain assessment

Ans: B, C, D, E Feedback: Vital signs, lung sounds, therapeutic effect including frequency of cough, and assessment of pain should be part of the ongoing client assessment. Auscultation of heart sounds is not part of the ongoing client assessment.

A nurse is developing a teaching plan about measures to reduce the risk of infection with Candida albicans from antiasthma therapy. The nurse includes this information because the client is prescribed which of the following? Select all that apply. A) Albuterol (Ventolin) B) Cromolyn (Gastrocrom) C) Fluticasone (Flovent) D) Tiotropium (Spiriva) E) Budesonide/formoterol (Symbicort)

Ans: B, C, E Feedback: Mast cell aerosols such as cromolyn and inhaled corticosteroids (ICSs) such as fluticasone and budesonide/formoterol have been associated with the development of oral thrush. Therefore, the client needs instructions on how to reduce his risk. Albuterol, a short-acting beta-2 agonist, and tiotropium, a cholinergic blocker, are not associated with the development of thrush.

The nurse instructs a client with asthma to always carry a rescue inhaler or quick-relief medication with him at all times. Which of the following would the client most likely carry? Select all that apply. A) Salmeterol (Serevent) B) Metaproterenol (Alupent) C) Tiotropium (Spiriva) D) Albuterol (Proventil) E) Formoterol (Foradil)

Ans: B, D Feedback: Short-acting beta agonists (SABAs) such as metaproterenol and albuterol are used as rescue treatment for asthma. Salmeterol and formoterol are long-acting beta agonists (LABAs). Tiotropium is a cholinergic blocking drug used to treat bronchospasm associated with COPD.

During stable chronic phases of asthma, the nurse should advise the client to monitor which of the following? Select all that apply. A) Blood pressure B) Wheezing C) Respiratory rate D) Coughing E) Peak flow changes

Ans: B, D, E Feedback: Clients with chronic stable asthma should monitor for symptoms such as wheezing, coughing, peak flow changes, and things that might be making the asthma worse.

A 25-year-old cystic fibrosis client presents to the clinic in obvious respiratory distress. Following physical exam, the health care provider suspects bronchiectasis based on which of the following findings? Select all that apply. A) Crushing, substernal chest pain B) Copious amounts of foul-smelling purulent sputum C) Neck vein distention D) Blood-tinged sputum E) Wheezing throughout the lung fields

Ans: B, D, E Feedback: Bronchiectasis is usually manifested by a chronic productive cough, often with several 100 mL of foul-smelling, purulent sputum a day. Hemoptysis is common. Dyspnea and wheezing occur in about 75% of clients. Crushing substernal chest pain and next vein distention are more suggestive of pulmonary emboli or myocardial infarction with right-sided heart failure.

When teaching a client about using a decongestant in a nasal spray form, which instruction would the nurse include? Select all that apply. A) "Recline on a bed and hang your head over the edge." B) "Sniff hard for a few minutes after administration." C) "Make sure the tip of the container is touching the nasal mucosa." D) "Do not share the container with anyone except family members." E) "Know that nasal burning or stinging may occur."

Ans: B, E Feedback: A client should be instructed to administer a nasal spray while sitting upright, not allow the tip of the container to touch the nasal mucosa, and to sniff hard for a few minutes after administration. The client may experience some burning or stinging after the administration of the nasal spray. The container should not be shared with anyone.

A client taking metoprolol (Lopressor) 50 mg one tablet twice daily begins taking over-the-counter pseudoephedrine. The nurse would assess for which of the following? Select all that apply. A) Hyperglycemic episode B) Hypertensive episode C) Rebound congestion D) Hypoglycemic episode E) Bradycardic episode

Ans: B, E Feedback: A client taking a beta-adrenergic blocker, such as metoprolol, and a decongestant, such as pseudoephedrine, may develop an initial hypertensive episode followed by a bradycardic episode.

A client with exercise-induced bronchospasm calls the physician's office to obtain a refill for his breathing medication. The client would most likely be prescribed which of the following? Select all that apply. A) Beclomethasone (Qvar) B) Levalbuterol (Xopenex) C) Theophylline (Theo-Dur) D) Mometasone (Asmanex) E) Bitolterol (Tornalate)

Ans: B, E Feedback: Beta-2 agonists, such as levalbuterol and bitolterol, are used to treat exercise-induced bronchospasm. Beclomethasone and mometasone are inhaled corticosteroids. Theophylline is a xanthine derivative.

Which of the following clients is at risk for developing a preventable disorder related to prolonged immobility? A) A middle-aged adult male diagnosed with bronchitis related to chronic smoking B) A young adult female diagnosed with sarcoidosis requiring corticosteroids to return her to remission C) A postsurgical client who is refusing to get out of bed and walk and will not wear those "uncomfortable elastic stocking" D) A sleep apnea client related to a history of smoking who utilizes a C-PAP machine every night at bedtime to maintain airway

Ans: C Feedback: A lack of mobility can result in secondary atelectasis (through incomplete lung expansion) and pulmonary embolism (from deep vein thrombosis). This is not the case with the other listed disorders of ventilation and gas exchange.

A nurse is assigned to care for a client with a nonproductive cough. The client has been prescribed codeine sulfate. The nurse understands that this drug is contraindicated in which client? A) Client with head injury B) Client with COPD C) Premature infant D) Clients with asthma

Ans: C Feedback: Codeine sulfate is contraindicated in premature infants. Codeine sulfate should be used cautiously in clients with head injury, COPD, and asthma.

As a result of hypoxemia and polycythemia, persons with chronic obstructive bronchitis are prone to: A) Breakdown of elastin B) Left-sided heart failure C) Pulmonary hypertension D) Expiratory airway collapse

Ans: C Feedback: Hypoxemia causes reflex vasoconstriction of the pulmonary vessels and further impairment of gas exchange in the lung. Hypoxemia also stimulates red blood cell production, causing polycythemia. As a result, persons with chronic obstructive bronchitis may develop pulmonary hypertension and right-sided heart failure. With breakdown and loss of lung elasticity and hyperinflation of the lungs with emphysema, the airways often collapse during expiration.

The nurse is reviewing the medication record of a client with asthma and notes that the health care provider has prescribed a monoclonal antibody as part of the treatment plan. The nurse would anticipate administering which drug? A) Zafirlukast B) Zileuton C) Omalizumab D) Salmeterol

Ans: C Feedback: Omalizumab (Xolair) is a monoclonal antibody used in the treatment of asthma. Leukotriene receptor antagonists include montelukast (Singulair) and zafirlukast (Accolate). Zileuton (Zyflo) is classified as a leukotriene formation inhibitor. Salmeterol (Serevent Diskus) is a long-term beta-2 agonist.

A 21-year-old client complains of a mild stinging sensation on using a nasal spray decongestant. Which response by the nurse would be most appropriate? A) "You need to stop the medication immediately." B) "The dose is probably too strong and needs to be reduced." C) "This sensation is common and usually disappears with continued use." D) "We better contact your primary health care provider right away."

Ans: C Feedback: The nurse needs to assure the client that the mild stinging sensation usually disappears with continued use. The drug needs to be stopped only if the stinging sensation is severe. The dose of the medication does not need to be altered. The primary health care provider does not need to be consulted immediately in the presence of a mild stinging sensation.

The nurse understands that an expectorant is administered cautiously to a client with which condition? A) Renal impairment B) Persistent headache C) Persistent cough D) Seizure disorder

Ans: C Feedback: The nurse should use the expectorant drugs with caution in clients with persistent cough. The nurse should use antitussives with caution in clients with persistent headache. The nurse needs to use opioid antitussives cautiously in clients with renal impairment and seizure disorders.

A client has been admitted to a health care facility with acute bronchospasm. The primary health care provider prescribes the drug epinephrine. The nurse anticipates administering this drug by which route? A) Intravenous B) Intramuscular C) Subcutaneous D) Intradermal

Ans: C Feedback: The nurse should use the subcutaneous route to administer epinephrine for acute bronchospasm. Doses of epinephrine are measured in tenths of a milliliter. A tuberculin syringe is used for measuring and administering these drugs by the parenteral route. The other routes are not appropriate for this situation.

A nurse is providing care to a client receiving theophylline. The client has received two loading doses and the nurse is evaluating the client's theophylline levels. Which finding would the nurse interpret as a therapeutic drug level? A) 5 mcg/L B) 8 mcg/L C) 13 mcg/L D) 20 mcg/L

Ans: C Feedback: Therapeutic theophylline levels range from 10 to 20 mcg/L. The possibility of toxicity increases with levels over 15 mcg/L, with toxicity indicated with levels over 20 mcg/L.

A client has just been admitted to the postsurgical unit following a below-the-knee amputation. Which of the following measures should her care team prioritize to prevent atelectasis during the client's immediate recovery? A) Bedrest and supplementary oxygen by nasal cannula B) Administration of bronchodilators by nebulizer C) Deep-breathing exercises and early mobilization D) Adequate hydration and a high-humidity environment

Ans: C Feedback: Coughing and deep breathing and early ambulation decrease the likelihood of atelectasis developing in surgical clients; bedrest should be avoided when possible. Oxygen, bronchodilators, hydration, and high humidity do not prevent atelectasis.

A client complains of increased sedation after the initiation of chlorpheniramine to treat her allergies. Which of the following would the nurse suggest to the client to treat her allergy symptoms that would result in less sedation? Select all that apply. A) Diphenhydramine (Benadryl) B) Clemastine (Tavist) C) Loratadine (Claritin) D) Cetirizine (Zyrtec) E) Phenylephrine (Neo-Synephrine)

Ans: C, D Feedback: Loratadine and cetirizine are second-generation antihistamines, which can be less sedating than first-generation antihistamines, like chlorpheniramine, diphenhydramine, and clemastine. Phenylephrine is a decongestant used to treat nasal congestion.

A client with a history of emphysema is experiencing hypoxemia after a taxing physical therapy appointment. Which of the following physiologic phenomena will occur as a consequence of hypoxemia? A) Peripheral vasodilation B) Necrosis C) Hypoventilation D) Increased heart rate

Ans: D Feedback: Consequences of hypoxemia include peripheral vasoconstriction, hyperventilation, and increased heart rate. Mild to moderate hypoxemia does not result in cell death and necrosis.

With acute respiratory distress syndrome (ARDS), a client progressively increases his work of breathing. The physiological principle behind this respiratory distress is related to: A) Increases in left atrial pressure causing thickening of the lining of the pulmonary arteries B) The elevation of pulmonary venous pressure C) Structural abnormalities of pulmonary vessels with proliferation of the vessel intima D) The stiffening of the lung, making it more difficult to inflate

Ans: D Feedback: As the disease (ARDS) progresses, the work of breathing becomes greatly increased as the lung stiffens and becomes more difficult to inflate. There is increased intrapulmonary shunting of blood, impaired gas exchange, and hypoxemia despite high supplemental oxygen therapy. With pulmonary hypertension, there are continued increases in left atrial pressure, which can lead to medial hypertrophy and intimal thickening of the small pulmonary arteries, causing sustained hypertension. Pulmonary hypertension also causes an elevation of pulmonary venous pressure.

A group of nursing students are reviewing information about leukotriene modifiers. The students demonstrate understanding of this class of drugs when they identify that these drugs are given by which route? A) Nebulization B) Metered-dose inhaler C) Nasal spray D) Orally

Ans: D Feedback: Leukotriene modifiers are only administered orally.

A car accident client is admitted with a chest tube following pneumothorax. He also has an elevated blood alcohol level. When the nurse enters his room, she notes the client is dyspneic, short of breath, and holding his chest tube in his hand. When the nurse pulls the linens back, she finds a "sucking" chest wound. After calling a "code blue," the next priority intervention would be to: A) Place the client's meal napkin over the wound B) Observe and wait for the code blue team to bring equipment C) Try to calm the patient down by maintaining therapeutic communication D) Apply a Vaseline gauze (airtight) dressing over the insertion site

Ans: D Feedback: The client has a medical emergency. Sucking chest wounds, which allow air to pass in and out of the chest cavity, should be treated by promptly covering the area with an airtight covering. Chest tubes are inserted as soon as possible. The other interventions will not help minimize the amount of air entering the pleural space.

A client with vasomotor rhinitis has been prescribed an antihistamine. The client is eager to know whether the prescribed antihistamine may cause excessive sedation. Which antihistamine would the nurse identify as having very little sedative effect? A) Brompheniramine B) Clemastine C) Chlorpheniramine D) Azelastine

Ans: D Feedback: The nurse should assure the client that azelastine has very little sedative effect; it is a second-generation antihistamine with little effect on central nervous system (CNS) depression. Brompheniramine, clemastine, and chlorpheniramine are first-generation antihistamines. Sedation is seen more often with first-generation antihistamines.

A nurse is caring for a 30-year-old client who is receiving albuterol for asthma. The client complains of feeling dizzy, especially when he stands up after sitting. The nurse suspects that a possible interaction with another drug could be causing the client's problem. Which of the following drugs should the nurse consider as a possible cause? A) Warfarin B) Uterine stimulants C) Methylxanthines D) Methyldopa

Ans: D Feedback: The nurse should consider methyldopa as a cause for the client's complaint of dizziness on standing, suggesting hypotension. Methyldopa and albuterol interact, leading to hypotension. Albuterol does not interact with warfarin. Interaction between albuterol and uterine stimulants leads to severe hypotension, and not severe headache. There is an increased risk of cardiotoxicity when methylxanthines are given along with albuterol.

A nurse is teaching a client who is prescribed albuterol about the adverse reactions associated with the drug. Which of the following symptoms, if experienced, should the nurse instruct the client to report to the health care provider? A) Fall in blood pressure B) Increased nighttime urination C) Hearing impairment or deficit D) Headache and flushing

Ans: D Feedback: The nurse should instruct the client to contact the health care provider if palpitations, tachycardia, chest pain, muscle tremors, dizziness, headache, flushing, or difficulty with urination or breathing occurs. Fall in blood pressure, increased nighttime urination, and hearing impairment are not adverse effects associated with a sympathomimetic bronchodilator.

A client with allergic rhinitis is prescribed an antihistamine. The nurse instructs the client to suck on sugarless hard candy to address which of the following? A) Drowsiness and sedation B) Thickening of the bronchial secretions C) Altered sensation of taste D) Dryness of the oral mucosa and the throat

Ans: D Feedback: The nurse should instruct the client to suck on a sugarless hard candy to prevent dryness of the oral mucosa and the throat, which is a side effect of antihistamine therapy. Sucking on candy does not relieve drowsiness, sedation, and thickening of the bronchial mucosa seen with antihistamine therapy. Altered sense of taste does not occur with most antihistamines.

A client is prescribed inhaled corticosteroid therapy along with bronchodilator therapy. Which of the following points should the nurse include in the teaching plan? A) "Stop corticosteroid therapy immediately if you notice any adverse effects." B) "Before each dose of corticosteroid, rinse the mouth thoroughly with water." C) "The corticosteroid drug provides rapid relief during an asthma attack." D) "Take the corticosteroid several minutes after the bronchodilator dose."

Ans: D Feedback: The nurse should instruct the client to take the corticosteroid several minutes after the bronchodilator dose. This helps in enhancing the application of the steroid into the bronchial tract. Corticosteroid therapy should never be stopped abruptly. The mouth should be rinsed thoroughly with water after each dose of corticosteroid to prevent the occurrence of fungal infections. The steroid drug does not provide rapid relief during an asthma attack, as it does not dilate the bronchus.

A 10-year-old child with asthma is prescribed high doses of an inhaled corticosteroid. The nurse would discuss which of the following with the child and parents as being affected? A) Blood pressure B) Skin turgor C) Urine output D) Rate of growth

Ans: D Feedback: The nurse should monitor the rate of growth of the child. Children are at risk for a reduction in growth when oral corticosteroids or higher doses of the inhalant form are used. Blood pressure, skin turgor, and urine output are not altered with corticosteroid therapy.

A nurse is caring for a client who is receiving cromolyn orally. Which of the following instructions would the nurse include in the client's teaching plan? A) Do not take the drug at bedtime. B) Swallow the drug without chewing. C) Take the drug with food or milk. D) Take the drug at least 30 minutes before meals.

Ans: D Feedback: When administered orally, cromolyn is given 30 minutes before meals and at bedtime. The oral form of the drug comes in an ampule. The ampule is opened and the contents are poured into a glass of water. The nurse stirs the mixture thoroughly. The client must drink all of the mixture. The drug may not be mixed with any other substance (e.g., fruit juice, milk, or foods).

A newly admitted critical head injury client presents to the neuro-ICU. The client is unresponsive to painful stimuli but able to breathe on his own. As the shift progresses, the nurses note a decrease in the client's respiratory effort. The client cannot maintain his O2 saturation above 70%. The nurses should anticipate assisting in beginning what type of pulmonary support? A) Increase oxygen level to 10 L/min. B) Begin Bi-PAP. C) Call respiratory therapy to suction the client. D) Prepare for mechanical ventilation.

Ans: D Feedback: When alveolar ventilation is inadequate to maintain PO2 or PCO2 levels because of respiratory or neurologic failure, mechanical ventilation may be lifesaving. Usually a nasotracheal, orotracheal, or tracheotomy tube is inserted into the trachea to provide the client with the airway needed for mechanical ventilation.

Which of the following manifestations typically accompanies an asthmatic attack? A) Decreased residual volume B) Decreased pulmonary arterial pressure C) Prolonged inspiration D) Hyperinflation of the lungs

Ans: D Feedback: During a prolonged attack, air becomes trapped behind the occluded and narrowed airways, causing hyperinflation of the lungs. This produces an increase in the residual volume of the lungs. Pulmonary arterial pressure tends to increase and expiration becomes prolonged.

A client is prescribed an antitussive for home use. Which of the following should a nurse include in the client's teaching plan? Select all that apply. A) Decreasing fluid intake during treatment with an antitussive B) Encouraging the use of sedatives during treatment C) Drinking fluids at least 30 minutes after taking a lozenge form D) Swallowing oral antitussive capsules whole E) Avoiding respiratory irritants during antitussive treatment

Ans: D, E Feedback: A client's antitussive teaching plan should include the following instructions: do not exceed recommended dose; avoid respiratory irritants; drink plenty of fluids if not contraindicated; swallow oral capsules whole; avoid drinking fluids for 30 minutes after taking a lozenge; do not use alcohol or other CNS depressants while being treated with antitussives; and contact the physician if cough is not relieved or becomes worse or is accompanied by chills, fever, chest pain, or sputum production.

The nurse is preparing to administer a drug that acts by reducing the swelling in the nasal passages by vasoconstriction. Which of the following might the nurse be administering? Select all that apply. A) Loratadine B) Guaifenesin C) Dextromethorphan D) Phenylephrine E) Oxymetazoline

Ans: D, E Feedback: Phenylephrine and oxymetazoline are decongestants that reduce swelling in the nasal passages by vasoconstriction. Loratadine is an antihistamine, guaifenesin is an expectorant, and dextromethorphan is a centrally acting antitussive.

A nurse is developing a teaching plan for a client who is to use a dry powder inhaler. Which of the following would the nurse include in the teaching plan? Select all that apply. A) Place device in water to clean. B) Swallow capsules provided. C) Hold inhaler 1 to 2 inches from mouth. D) Hold breath for 10 seconds. E) Inhale quickly.

Ans: D, E Feedback: To properly use a dry powder inhaler, the client should do the following: prepare the medication for inhalation, place the mouthpiece close to the lips, inhale quickly, hold breath for 10 seconds, not swallow capsules provided, and not place the inhaler in water.


Conjuntos de estudio relacionados

Data Analytics - INFOU796 - Chapter 2 Quiz

View Set

ch 28-29 APUSH The American Pageant Rogan

View Set

Chapter 2: Traditional and Contemporary Management Perspectives

View Set

FINAL EXAM Supporting Client Operating Systems

View Set

Words that mean the opposite when using the prefix in/im or ex.

View Set

Types of Medication Distribution Systems

View Set